You are on page 1of 66

Answer Key

1. (b) 11. (a) 21. (c) 31. (a) 41. (b) 51. (a) 61. (a) 71. (a) 81. (a) 91. (d)
2. (c) 12. (a) 22. (d) 32. (a) 42. (a) 52. (c) 62. (c) 72. (a) 82. (b) 92. (d)
3. (b) 13. (b) 23. (b) 33. (c) 43. (d) 53. (c) 63. (d) 73. (d) 83. (b) 93. (c)
4. (c) 14. (c) 24. (b) 34. (b) 44. (c) 54. (d) 64. (a) 74. (c) 84. (c) 94. (d)
5. (b) 15. (d) 25. (c) 35. (c) 45. (c) 55. (b) 65. (c) 75. (b) 85. (c) 95. (b)
6. (d) 16. (b) 26. (c) 36. (c) 46. (d) 56. (b) 66. (b) 76. (b) 86. (c) 96. (b)
7. (b) 17. (a) 27. (b) 37. (d) 47. (a) 57. (c) 67. (d) 77. (b) 87. (b) 97. (c)
8. (a) 18. (d) 28. (a) 38. (b) 48. (c) 58. (c) 68. (b) 78. (b) 88. (c) 98. (c)
9. (b) 19. (c) 29. (d) 39. (c) 49. (d) 59. (a) 69. (b) 79. (c) 89. (a) 99. (c)
10. (c) 20. (a) 30. (a) 40. (b) 50. (b) 60. (a) 70. (a) 80. (a) 90. (c) 100. (c)

Q1.
Answer: b
Explanation:
● While the Constitution of India does not explicitly outline the concept of natural justice, it is widely
regarded as an indispensable component in ensuring the fairness of the judicial process. Originating
from the principles of 'Jus Natural', which pertains to natural law, natural justice embodies a
fundamental understanding of morality and fairness. In its layman's language, natural justice means a
natural sense of what is right and wrong and in its technical sense, it is synonymous with fairness.
● Natural justice implies fairness, reasonableness, equity and equality. Natural Justice is a concept of
Common Law and it is the counterpart of the American concept of ‘procedural due process’. Natural
justice represents higher procedural principles developed by judges which every administrative agency
must follow in taking any decision adversely affecting the rights of a private individual.
Therefore, option (b) is the correct answer.
Knowledge Box
● The Principle of Proportionate Justice states that justice in some cases would mean
rewarding people in proportion to the scale and quality of their effort.
● Distributive justice concerns the fair allocation of resources among diverse members
of a community. The principle says that every person should have access to
approximately the same level of material goods and services.
● Retributive justice is defined as a form of justice that focuses on the punishment of
the offender and not on rehabilitation. This approach is centered around ensuring that
the offender faces consequences for their actions, rather than primarily aiming to assist
them in their personal growth or viewing them as individuals who have erred.
Relevance: The Chief Justice of India has said that the sealed cover practice infringes the core of the principles
of natural justice.

1
Vajiram & Ravi Prelims (2024)
PowerUp All India Mock Test - 01
Q2.
Answer: c
Explanation:
A State law reserving a portion of private sector jobs for the domicile of the concerned State violates the
following Articles of the Constitution of India:
● Article 16: It provides for equality of opportunity for all citizens in matters of employment or
appointment to any office under the State. No citizen can be discriminated against or be ineligible for
any employment or office under the State on grounds of only religion, race, caste, sex, descent, place of
birth or residence.
○ Article 16 (3): Nothing in this article shall prevent Parliament from making any law prescribing,
in regard to a class or classes of employment or appointment to an office under the Government
of, or any local or other authority within, a State or Union Territory, any requirement as to
residence within that State or Union Territory prior to such employment or appointment. So,
point 1 is correct.
● Article 19 (1) (g): All citizens are given the right to practise any profession or to carry on any occupation,
trade or business. This right is very wide as it covers all the means of earning one’s livelihood. So, point
2 is correct.
● Article 35: It states that the Parliament shall have, and the Legislature of a State shall not have, power
to make laws with respect to any of the matters which under clause (3) of Article 16, clause (3) of Article
32, Article 33 and Article 34 may be provided for by law made by Parliament. So, point 4 is correct.
○ Article 16 states that the Parliament can enact any legislation requiring residence in a State or
Union Territory as a pre-condition for particular employment or appointments in the respective
State or Union Territory or in local authorities or other authorities within that State or Union
Territory.
● Article 17: It abolishes ‘untouchability’ and forbids its practice in any form. The enforcement of any
disability arising out of untouchability shall be an offence punishable in accordance with law. It is not
violated when a State law reserving a portion of private sector jobs for the domicile of the concerned
State. So, point 3 is not correct.
Therefore, option (c) is the correct answer.
Relevance: The Punjab and Haryana High Court recently quashed a law enacted by the Haryana Government in
2021 that guaranteed 75% reservation to locals in private sector jobs in Haryana.

Q3.
Answer: b
Explanation:
● Article 3 of the Constitution of India authorises the Parliament to:
○ form a new State by separation of territory from any State or by uniting two or more States or
parts of States or by uniting any territory to a part of any State;
○ increase the area of any State;
○ diminish the area of any State;
○ alter the boundaries of any State; and
○ alter the name of any State.
● However, Article 3 lays down two conditions in this regard: one, a Bill contemplating the above changes
can be introduced in the Parliament only with the prior recommendation of the President; and two,
before recommending the Bill, the President has to refer the same to the State Legislature concerned
for expressing its views within a specified period. So, statements 1 and 2 are correct.

2
Vajiram & Ravi Prelims (2024)
PowerUp All India Mock Test - 01
○ The President (or Parliament) is not bound by the views of the State Legislature and may either
accept or reject them, even if the views are received in time. Further, it is not necessary to make
a fresh reference to the State Legislature every time an amendment to the Bill is moved and
accepted in Parliament. In case of a Union Territory, no reference need be made to the concerned
legislature to ascertain its views and the Parliament can itself take any action as it deems fit.
● The decision of the Central Government to cede part of a territory known as Berubari Union (West
Bengal) to Pakistan led to political agitation and controversy and thereby necessitated the Presidential
reference. The Supreme Court held that the power of Parliament to diminish the area of a State (under
Article 3) does not cover cession of Indian territory to a foreign country. Hence, Indian territory can be
ceded to a foreign state only by amending the Constitution under Article 368. Consequently, the 9th
Constitutional Amendment Act (1960) was enacted to transfer the said territory to Pakistan. On the other
hand, the Supreme Court in 1969 ruled that settlement of a boundary dispute between India and
another country does not require a Constitutional Amendment. It can be done by executive action as it
does not involve cession of Indian territory to a foreign country. So, statement 3 is not correct.
So, only two of the above statements are correct.
Therefore, option (b) is the correct answer.
Relevance: Recently the Supreme Court has upheld the abrogation of Article 370 of the Constitution. In this
background, the procedure for reorganization of states has gained spotlight.

Q4.
Answer: c
Explanation:
● Charter Act of 1793: The Governor General's and Governors' powers to overrule their council were
emphasized and explained. This power had been given specially to Governor General Cornwallis in 1786.
Governor General's control over the Presidencies was strengthened. He was allowed to issue orders
and directions to any Government and Presidency of lndia during his absence from Bengal without
previous consultation with his council. He could exercise all executive power vested in the Central
Government.
● A regular code of all regulations that could be enacted for the internal Government of the British
territory in Bengal was framed. The regulation applied to the rights, persons and property of the Indian
people and it bound the Courts to regulate their decisions by the rules and regulations contained therein.
It also required that, "all laws relating to the rights of the person and property should be printed with
translation in Indian languages and prefixed with statements of grounds on which they were enacted,
"so that the people should become familiar with their rights. privileges and immunities.
● The Act of 1793 thus laid the foundation of government by written laws and regulations in British lndia
in place of the personal rule of the past rulers. The interpretation of regulations and written laws was
to be done by the Courts. The concepts of a civil law enacted by a secular human agency and applied
universally, was an important change.
Therefore, option (c) is the correct answer.

3
Vajiram & Ravi Prelims (2024)
PowerUp All India Mock Test - 01
Q5.
Answer: b
Explanation:
● Article 74 of the Constitution of India provides for a Council of Ministers (CoM) with the Prime Minister
at the head to aid and advise the President in the exercise of his functions. However, the President may
require the CoM to reconsider such advice and the President shall act in accordance with the advice
tendered after such reconsideration. The advice tendered by CoM to the President shall not be inquired
into in any court. This provision emphasises the intimate and the confidential relationship between the
President and the Ministers. So, point 1 is correct.
● According to Article 110 (3) of the Constitution of India, if any question arises whether a Bill is a Money
Bill or not, the decision of the Speaker of the House thereon shall be final. But the Constitution of India
does not explicitly prohibit courts from conducting inquiries. The Supreme Court in the 2018 Aadhaar
case had said that the Speaker’s decision will be subject to judicial scrutiny. So, point 2 is not correct.
● Article 122 of the Constitution provides that the Courts cannot inquire into proceedings of Parliament.
The validity of any proceedings in Parliament shall not be called in question on the ground of any alleged
irregularity of procedure. No officer or Member of Parliament in whom powers are vested by or under
this Constitution for regulating procedure or the conduct of business, or for maintaining order, in
Parliament shall be subject to the jurisdiction of any court in respect of the exercise by him of those
powers. So, point 3 is correct.
So, in only two of the above cases, the Constitution of India explicitly prohibits enquiry by courts.
Therefore, option (b) is the correct answer.

Q6.
Answer: d
Explanation:
● The Preamble to the Constitution of India is a reflection of the core constitutional values that embody
the Constitution. It declares India to be a Sovereign, Socialist, Secular Democratic, Republic committed
to Justice, Equality and Liberty for the people.
● It does not grant substantive rights and is not enforceable in the courts, but various Indian courts have
engaged with the Preamble and have treated it as a guiding light in the interpretation of the Constitution.
● In the Keshavananda Bharati case (1973), the Supreme Court held that the Preamble to the Constitution
of India is a part of the constitution and is neither a source of power nor a source of limitation. It further
observed that the Constitution should be read and interpreted in the light of the grand and noble vision
expressed in the Preamble.
Therefore, option (d) is the correct answer.
Relevance: Recently, about 2.27 crore people registered for the reading of the Preamble of the Indian
Constitution in Karnataka on the ‘International Day of Democracy’.

4
Vajiram & Ravi Prelims (2024)
PowerUp All India Mock Test - 01
Q7.
Answer: b
Explanation:
● Parliament has recently passed the 'Chief Election Commissioner and other Election Commissioners
(Appointment, Conditions of Service and Term of Office) Bill, 2023'. The Bill replaces the 'Election
Commission (Conditions of Service of Election Commissioners and Transaction of Business) Act, 1991'. It
provides for the appointment, salary, and removal of the Chief Election Commissioner (CEC) and Election
Commissioners (ECs). According to the provisions of the Bill, the CEC and ECs will be appointed by the
President upon the recommendation of a Selection Committee. The Selection Committee will consist of
the Prime Minister, a Union Cabinet Minister, and Leader of Opposition or leader of the largest
opposition party in the Lok Sabha. So, point 1 is not correct.
● The Central Vigilance Commission is a multi-member body consisting of a Central Vigilance
Commissioner (chairperson) and not more than two vigilance commissioners. They are appointed by the
President by warrant under his hand and seal on the recommendation of a three member committee
consisting of the Prime Minister as its head, the Union Minister of Home Affairs and the Leader of the
Opposition in the Lok Sabha. They hold office for a term of four years or until they attain the age of sixty
five years, whichever is earlier. After their tenure, they are not eligible for further employment under
the Central or a State Government. So, point 2 is not correct.
● The Central Bureau of Investigation (CBI) is governed by The Delhi Special Police Establishment (DSPE)
Act, 1946. The Act says “the Central Government shall appoint the Director on the recommendation of
the Committee consisting of the Prime Minister as the chairperson, the Leader of Opposition in the Lok
Sabha or, where there is no such Leader of Opposition, the Leader of the single largest Opposition Party,
and the Chief Justice of India or Judge of the Supreme Court nominated by him”. So, point 3 is correct.
● The Lokpal chairperson and its members are appointed by the President after obtaining the
recommendations of a selection committee headed by the Prime minister and comprising the Speaker
of the Lok Sabha, the Leader of Opposition in the Lower House, the Chief Justice of India or a Judge of
the Supreme Court nominated by him, and an eminent jurist as recommended by the chairperson and
members of the selection panel. So, point 4 is correct.
So, the Chief Justice of India is a member of the selection panel for only two of the above appointments.
Therefore, option (b) is the correct answer.
Relevance: Recently, Parliament has passed the Chief Election Commissioner and other Election Commissioners
(Appointment, Conditions of Service, and Term of Office) Bill, 2023.

Q8.
Answer: a
Explanation:
Article 85(1): It empowers the President to summon each House of Parliament to meet at such time and place
as he thinks fit, but six months shall not intervene between its last sitting in one Session and the date appointed
for its first sitting in the next Session. The President exercises the power to summon the Houses on the
recommendation of the Prime Minister or the Cabinet.
● The decision to summon, prorogue, or dissolve the Lok Sabha is typically made by the President on the
advice of the Prime Minister and the Council of Ministers. The Prime Minister alone doesn’t possess the
authority to take these actions without the Cabinet's involvement. So, statement 1 is not correct.

5
Vajiram & Ravi Prelims (2024)
PowerUp All India Mock Test - 01
● It is not provided by the Constitution of India that Lok Sabha and Rajya Sabha should be summoned and
prorogued simultaneously or on the same dates. Until 1961, Sessions of the two Houses generally
commenced on different dates, except for the first Session of every year when the President addresses
members of both the Houses assembled together. However, since 1962, generally both the Houses have
been meeting simultaneously. So, statement 2 is not correct.
● Articles 86 and 87 of the Constitution deal with the Address by the President. Article 86 confers a right
on the President to address either the House of Parliament or both Houses assembled together, and
for that purpose require the attendance of members. However, since the commencement of the
Constitution, the President has not so far addressed a House or Houses together under this provision.
So, statement 3 is correct.
● Article 87 deals with Special Address by the President and provides that the President shall address both
Houses of Parliament assembled together at the commencement of the first session after each general
election to the Lok Sabha and at the commencement of the first session of each year and inform
Parliament of the causes of its summons.
So, only one of the above statements is correct.
Therefore, option (a) is the correct answer.
Relevance: Recently, there have been speculations that the current government may dissolve Parliament and
call early elections.

Q9.
Answer: b
Explanation:
Under the Constitution of India, the Parliament consists of three parts viz, the President, the Council of States
and the House of the People.
● Article 83 (2): It states that the normal term of the Lok Sabha is five years from the date of its first
meeting after the general elections, after which it automatically dissolves.
● However, the President is authorized to dissolve the Lok Sabha at any time even before the completion
of five years and this cannot be challenged in a court of law.
● Further, the term of the Lok Sabha can be extended during the period of National Emergency by a law
of Parliament for one year at a time for any length of time. However, this extension cannot continue
beyond a period of six months after the emergency has ceased to operate.
Therefore, option (b) is the correct answer.
Relevance: The Lok Sabha has functioned for 33% of its scheduled time which suggests that the 17th Lok Sabha
could be the shortest full-term since 1952.

Q10.
Answer: c
Explanation:
● Representation of the People Act, 1951 has laid down certain conditions for a political party to form
and get registered by the Election Commission of India (ECI). These are:
○ It must consist only of Indian citizens. So, statement 1 is correct.
○ It must have at least 100 registered electors as its members. So, statement 2 is correct.
○ It must call itself a political party set up for the purpose of contesting elections to the Parliament
and State Legislatures and for no other purpose. So, statement 4 is correct.

6
Vajiram & Ravi Prelims (2024)
PowerUp All India Mock Test - 01
● Presently, there is no statutory backing for internal democratic regulation of political parties in India
and the Election Commission of India’s (ECI) power to require parties to hold regular internal elections
for office bearers and candidate selection is compromised in the absence of any penal provisions. So,
statement 3 is not correct.
Therefore, option (c) is the correct answer.
Knowledge Box

Can political parties be de-registered by the Election Commission of India?


● The ECI is not empowered to de-register parties on the grounds of violating the
Constitution or breaching the undertaking given to it at the time of registration.
● A party can only be de-registered if its registration was obtained by fraud; if it is
declared illegal by the Central Government; or if a party amends its internal
Constitution and notifies the ECI that it can no longer abide by the Indian Constitution.
Relevance: Recently, the Election Commission of India is aiming to push for internal democracy within political
parties.

Q11.
Answer: a
Explanation:
● Money multiplier shows the mechanism by which reserve money creates money supply in the economy.
A country's money multiplier depends on two factors—how much individuals (and businesses) hold in
cash and how much banks hold as reserves. The more individuals hold cash in hand, the less the banking
system will be able to create money and hence a lower value for the multiplier. In other words, cash
in hand acts as a leakage for the banking system.
● On the other hand, if people start holding more currency in bank deposits and less in hand, this will lead
to an increase in reserves of the commercial banks. Hence, a higher value for the multiplier. Increase
in lendable resources will also result in decrease in cost of credit for borrowers.
● A higher value for this ratio, called the money multiplier, indicates that the banking system generates a
higher money supply out of money given by the Central Bank.
○ In India, the recent push to financial inclusion has led to people holding less cash in hand (relative
to deposits) leading to an increase in the money multiplier.
● Every time a Rupee is deposited into a bank account, a bank's total reserves increases. The bank will
keep some of it on hand as required reserves, but it will loan the excess reserves out. When that loan is
made, it increases the money supply.
Therefore, option (a) is the correct answer.
Knowledge Box

● The tax multiplier in economics is defined as the factor by which a change in taxes will
alter GDP. With this tool, the government is able to decrease (increase) taxes by the
exact amount that they need for GDP to rise (decline). This allows the government to
make a precise tax change rather than an estimation.
Relevance: Recently, the State Bank of India (SBI) published research outlining how the recent withdrawal of
rupees two thousand notes may increase bank deposits, loan repayment, and even the nation's GDP.

7
Vajiram & Ravi Prelims (2024)
PowerUp All India Mock Test - 01
Q12.
Answer: a
Explanation:
A stronger US dollar tends to inflate India’s import bill and widen the deficit between its imports and exports.
This is because India relies on US dollar-denominated imports for over 85 per cent of its crude oil requirements
and imports more goods than it exports. Therefore, India’s import bill usually shoots up when the dollar
strengthens, increasing the domestic demand for dollars. So, statements 1 and 2 are correct.
● When the value of US dollar strengthens, the Foreign Portfolio Investments (FPI) is pulled out from the
Indian market due to lower demand for Indian Rupee and higher demand for US dollar. This worsens the
situation further by increasing the domestic demand for dollars.
So, both Statement–I and Statement–II are correct and Statement–II is the correct explanation for Statement–
I.
Therefore, option (a) is the correct answer.
Knowledge Box

● When the rupee slides against the dollar, the Reserve Bank of India (RBI) has two main
instruments to stem the slide.
○ It can put through sharp interest rate hikes in India to make domestic bonds
and gilts more attractive to foreign investors, so that they rethink their pullouts.
○ It can use its large foreign exchange reserves to intervene directly in the
currency market. It can also sell dollars in the futures and forward markets.
When RBI sells dollars, it results in extinguishing an equivalent amount in
rupees, thus reducing the rupee liquidity in the system. Less Indian Rupee in the
market will appreciate its value against the US dollar.
Relevance: For over a year, the U.S. dollar has been consistently strengthening against various currencies.

Q13.
Answer: b
Explanation:
● The Reserve Bank of India (RBI) is responsible for issuing notes of all denominations except one Rupee
note. Thus, RBI is not empowered to issue bank notes of denomination of value of one rupee. The
Government of India is solely responsible for the designing and minting of coins in various denominations
under the Coinage Act, 2011. The Union Ministry of Finance is responsible for the circulation of one
rupee note. The coins are issued for circulation only through RBI under the RBI Act, 1934.
● The Minimum Reserve System is the currency issue system followed by the RBI at present. It was adopted
in 1956. The Minimum Reserve System which requires the RBI to keep a minimum reserve of Rs 200
crores comprising foreign currencies, gold coin and gold bullion (minimum of Rs 115 crore in the form of
gold). There is no limit for the RBI to issue currencies by keeping this minimum reserve. So, statement
1 is correct.
● State Government transactions can be carried out by the RBI in terms of the agreement entered into
with the State Governments under the RBI Act, 1934. As of now, such agreements exist between RBI and
all the State Governments except the Government of Sikkim. Thus, the legal provisions vest the Reserve
Bank of India with both the right and obligation to function as a banker to the government. So, statement
2 is not correct.

8
Vajiram & Ravi Prelims (2024)
PowerUp All India Mock Test - 01
● The RBI is the aggregator for accounting of all GST collections in the respective government accounts.
Agency banks who collect the GST for challans generated by taxpayers online on the GST portal report
the collections for settlement to government accounts to the RBI. The RBI has also facilitated the
payment of GST by taxpayers directly into government accounts at the RBI by using NEFT/RTGS payment
options provided in the GST portal. So, statement 3 is correct.
So, only two of the above statements are correct.
Therefore, option (b) is the correct answer.
Relevance: The Reserve Bank of India (RBI) has recently announced that over 97 per cent of circulating ₹2000
banknotes have returned till November 2023.

Q14.
Answer: c
Explanation:
● Foreign Portfolio Investment (FPI) is a type of investment made by foreign investors in the securities of
a country other than their own.
● It is one of the easiest methods of investing abroad. Unlike Foreign Direct Investment (FDI), which
involves long-term investments in physical assets, FPI involves the purchase of securities such as stocks,
bonds and mutual funds.
○ Additionally, the most important type of investment by FPI is in shares or equities due to the
strong performance of Indian equities.
● FPI is part of a country's capital account and is shown on its Balance of Payments (BOP). In India, FPI is
regulated by the Securities and Exchange Board of India (SEBI).
● According to the Reserve Bank of India (RBI) guidelines, there are many different instruments of FPI that
include:
○ Stocks: They are shares of ownership in a company. When one buys a stock, they are essentially
buying a piece of the company. So, point 1 is correct.
○ Bonds: They are loans that you make to a company or government. When you buy a bond, you
are lending money to the issuer and they agree to pay you back with interest over a set period of
time.
○ Mutual Funds: They are baskets of stocks or bonds that are managed by a professional. Mutual
funds offer a way for investors to diversify their portfolios and invest in a variety of assets without
having to do the research themselves. So, point 2 is correct.
○ Indian Depositary Receipts (IDRs): They are similar to GDRs (Global Depository Receipts). An FPI
may purchase, hold, or sell Indian Depository Receipts (IDRs) of companies resident outside India
and issued in the Indian capital market. So, point 4 is correct.
○ Exchange-Traded Funds (ETFs): They are similar to mutual funds, but they are traded on
exchanges like stocks. This makes them more liquid than mutual funds, which can be an
advantage for investors who want to be able to buy and sell their investments quickly. FPI may
trade or invest in all exchange-traded derivative contracts approved by the SEBI. So, point 5 is
correct.
● Note: Sovereign Gold Bonds (SGBs) are government securities denominated in grams of gold. They are
substitutes for holding physical gold. Investors have to pay the issue price in cash and the bonds will be
redeemed in cash on maturity. The Bond is issued by the Reserve Bank on behalf of the Government of
India.

9
Vajiram & Ravi Prelims (2024)
PowerUp All India Mock Test - 01
○ Only Persons resident in India as defined under the Foreign Exchange Management Act, 1999 are
eligible to invest in SGB. Eligible investors include individuals, HUFs, trusts, universities and
charitable institutions. FPIs are not eligible to invest in SGB. So, point 3 is not correct.
So, only four of the above instruments are eligible to be classified as Foreign Portfolio Investments in India.
Therefore, option (c) is the correct answer.
Relevance: Foreign Portfolio Investors (FPIs) have pumped in ₹21,350 crore in the primary market from April to
August in 2023.

Q15.
Answer: d
Explanation:
● Entrenched inflation implies that the rate of prices will not slow and the price rise in the economy will
persist for a long period with risk becoming permanent.
● It occurs when consumers’ expectations for inflation rise and price increase hit “sticky” categories like
rent or medical services where they aren’t likely to come down soon. Inflation persists for so long that
the expectation that it would continue to do so is firmly held by the people.
● When this happens, it can lead to a wage-price spiral where workers demand wage increases to preserve
their incomes as the cost of living soars. This then increases operational costs for companies, which, in
turn, raise their prices to maintain profitability, thereby exacerbating inflation.
● Stock prices are unlikely to rise in case of entrenched inflation. It is because the strategy and focus of
companies shift from growth to cost-cutting.
Therefore, option (d) is the correct answer.
Knowledge Box

● Bottleneck Inflation takes place when the supply falls drastically and the demand
remains at the same level. Such situations arise due to supply-side hurdles, hazards or
mismanagement which is also known as ‘structural inflation’. This could be put in the
‘demand-pull inflation’ category.
● Stagflation is the combination of slow economic growth, high unemployment and a
high rate of inflation.
● Disinflation is a decline in the rate of increase in the general price level of goods and
services in the Gross Domestic Product (GDP) of a country over time.
Relevance: The Economic Survey 2022-23 had highlighted the risks of entrenched inflation.

Q16.
Answer: b
Explanation:
● Balance of Payments account include current account, capital account and official reserves account.
● Current account records receipt and payment of foreign exchange on account of such transactions which
do not impact asset-liability status of a country in relation to the rest of the world. Liabilities or assets
of a country (in relation to the rest of the world) are neither raised nor reduced. In other words, current
account transactions do not give rise to 'future claims'. It includes transactions involving export and
import of goods, export and import of services and current transfer

10
Vajiram & Ravi Prelims (2024)
PowerUp All India Mock Test - 01
● Capital account records receipts and payments of such transactions which cause an impact on asset-
liability status of a country in relation to the rest of the world. Liabilities or assets of a country (in relation
to the rest of the world) are either raised or reduced. In other words, capital account transactions lead
to future claims. Two principal components of capital account are borrowing and foreign investment
(both Portfolio Investment and Foreign Direct Investment).
● Since sale of shares to a foreign investor affects the asset-liability status and gives rise to future claims,
it will be reflected only in the Capital account and not in the Current account of Balance of Payment.
Therefore, option (b) is the correct answer.

Q17.
Answer: a
Explanation:
● Minimum Support Price (MSP) is the minimum price set by the Government of India for buying certain
agricultural products directly from the farmers if the open market prices are less than the cost incurred.
● Based on recommendations of the Commission for Agricultural Costs and Prices (CACP), the
Department of Agriculture and Co-operation declares MSP for certain crops before their sowing seasons.
The CACP calculates this price based on the cost of cultivation and adds a markup.
● The CACP projects three kinds of production costs for every crop, both at State and all-India average
levels:
○ A2 covers all paid-out costs directly incurred by the farmers in cash and kind on seeds, fertilisers,
pesticides, hired labour, leased-in land, fuel, irrigation, etc. So, point 2 is correct.
○ A2+FL includes A2 plus an imputed value of unpaid family labour. So, point 1 is correct.
○ C2 is a more comprehensive cost that factors in rentals and interest forgone on owned land and
fixed capital assets respectively, on top of A2+FL. So, points 3 and 4 are not correct.
● CACP considers both A2+FL and C2 costs while recommending MSP. However, CACP reckons only A2+FL
cost for return (for providing MSP). The Central Government maintains that the MSP was fixed at a level
of at least 1.5 times of the all-India weighted average Cost of Production (CoP), but it calculates this cost
as 1.5 times of A2+FL.
○ C2 costs are used by CACP as benchmark opportunity costs to check if the MSPs recommended
by them at least cover these costs in some of the major producing States.
Therefore, option (a) is the correct answer.
Knowledge Box

Determinants of Minimum Support Price (MSP):


● Demand and supply
● Cost of production
● Price trends in the domestic and international market
● Inter-crop price parity
● Terms of trade between agriculture and non-agriculture
● Minimum of 50 per cent as the margin over cost of production
● Likely implications of MSP on consumers of that product
Relevance: Recently, few States have been assured of realising Minimum Support Price (MSP) in line with the
Swaminathan formula, also known as C2 + 50 per cent.

11
Vajiram & Ravi Prelims (2024)
PowerUp All India Mock Test - 01
Q18.
Answer: d
Explanation:
● The Special Drawing Right (SDR) is an interest-bearing international reserve asset created by the IMF in
1969 to supplement other reserve assets of member countries. SDR allocation is not a loan from the
IMF. It is not a currency, nor a claim on the IMF, but is potentially a claim on freely usable currencies of
IMF members. So, statement 1 is correct.
○ A basket of currencies defines the SDR: the US dollar, Euro, Chinese Yuan, Japanese Yen and the
British Pound.
● IMF members – and the IMF itself – hold SDRs and the IMF has the authority to approve other holders,
such as Central Banks and multilateral development banks, while individuals and private entities cannot
hold SDRs. So, statement 2 is correct.
● Countries can exchange their SDRs for hard currencies with other IMF members. An SDR allocation is a
way of supplementing Fund member countries’ foreign exchange reserves, allowing members to reduce
their reliance on more expensive domestic or external debt for building reserves. So, statement 3 is
correct.
● General allocations of SDRs are distributed across the IMF membership in proportion to IMF quota
shares. So, statement 4 is correct.
○ An individual member country’s quota broadly reflects its relative position in the world economy.
○ On this basis, the share of emerging markets and developing countries is about 42.3 per cent
(about USD 275 billion), of which 3.3 per cent (about USD 21 billion) is for low-income countries.
○ As of 2022, India's quota in the IMF, which determines voting shares in the multilateral lending
agency, is 2.75 per cent.
So, all four of the above statements are correct.
Therefore, option (d) is the correct answer.
Relevance: A Reserve Bank-appointed committee recently suggested a host of short-term and long-term
measures for internationalisation of the Indian rupee, including efforts for inclusion of the Indian currency in
IMF’s Special Drawing Rights (SDR) basket.

Q19.
Answer: c
Explanation:
● Goods and Services Tax (GST) is a comprehensive indirect tax levied on the supply of goods and services
in India. However, there are some commodities on which GST is not levied, either because they are
exempted from GST or because they fall outside the scope of GST altogether.
○ Judicial Stamp Paper is not taxable under GST, as it is subject to stamp duty levied by the State
Governments. So, point 4 is correct.
○ Electricity is exempted from GST as per the Notification 02/2017 (Central Tax) dated 28 June
2017 under the heading “Electrical Energy”. Further, services provided by an electricity
transmission or distribution utility by way of transmission or distribution of electricity are also
exempted from GST. So, point 5 is correct.
● Following items are included under the coverage of GST:
○ Online gaming is subject to a 28% GST rate on the full face value. It is imposed on entry-level
and not on winnings. The tax came into effect from October 1, 2023 and will be reviewed six
months after its implementation. So, point 1 is not correct.

12
Vajiram & Ravi Prelims (2024)
PowerUp All India Mock Test - 01
○ The GST Council levies a 5% GST on pre-packaged and labelled food items. Agricultural products
with pre-packaged labels like paneer, buttermilk, packaged curd, wheat flour, other cereals,
honey, papad, food grains, meat and fish (except frozen), puffed rice and jaggery, etc. are taxed
under GST. So, point 2 is not correct.
○ Insulin and oral rehydration salts are taxed at 5% under GST, as they are life-saving drugs or
medicines. GST on human blood and its by-products used in medicine and all types of
contraceptive goods are not levied. So, point 3 is not correct.
Therefore, option (c) is the correct answer.
Relevance: The Central Board of Indirect Taxes and Customs has implemented a 28 per cent Goods and Service
Tax on online gaming from October 1, 2023.

Q20.
Answer: a
Explanation:
● Ecological amplitude (Ecological Valence) is the degree of adaptation of a living organism to
environmental changes. This ecological amplitude is expressed quantitatively as the range of
environmental changes within which a given species can carry on its normal vital activities.
● Ecological amplitude can be examined as either the reaction of a species to individual environmental
factors or to an aggregate of factors. In the first case, species that can tolerate a wide range of changes
in the strength of an acting factor are designated by a term consisting of the name of the given factor
and the prefix “eury”.
○ eurythermal (referring to the effects of temperature), euryhaline (salinity), and eurybathic
(depth).
● Species that are adapted to only a narrow range of changes in a given factor are designated by the same
term with the prefix “steno”, for example, stenothermal and stenohaline.
○ Species that exhibit broad ecological amplitude for an aggregate of factors are called eurybionts,
while species with low adaptability are called stenobionts.
Therefore, option (a) is the correct answer.
Knowledge Box

● The carrying capacity of an ecosystem is the size of the population that can be
supported indefinitely upon the available resources and services of that ecosystem. It
also means that all plants and animals which an area of the Earth can support at once.

Q21.
Answer: c
Explanation:
Echolocation:
● It is a technique used by some animals to determine the location of objects using reflected sound. It
occurs when an animal emits a sound wave that bounces off an object, returning an echo that provides
information about the object’s distance and size. This allows the animals to move around in pitch
darkness so they can navigate, hunt, identify friends and enemies and avoid obstacles.
● Bats, whales, dolphins, a few birds like the nocturnal oilbird and some swiftlets, some shrews and the
similar tenrec from Madagascar are all known to echolocate. So, points 1, 2, 3 and 4 are correct.

13
Vajiram & Ravi Prelims (2024)
PowerUp All India Mock Test - 01
○ For dolphins and toothed whales, this technique enables them to see in muddy waters or dark
ocean depths and may even have evolved so that they can chase squid and other deep-diving
species.
○ Echolocation allows bats to fly at night as well as in dark caves. The oilbird is active at night, and
some insect-eating swiftlets roost in dark caves, so they have evolved the ability to echolocate.
Both bats and oilbird use sharp, audible clicks to navigate through the darkness.
● Bees do not use echolocation as a means of navigation or communication. They primarily rely on their
vision and other sensory mechanisms for navigation. They can navigate by their sense of smell, the sun,
the sky's pattern of polarized light, vertical landmarks that stand out from the panorama, and possibly
the Earth's magnetic field. So, point 5 is not correct.
So, only four of the above animals use echo-location to navigate.
Therefore, option (c) is the correct answer.
Knowledge Box

● Elephants cannot use echolocation. While elephants have excellent hearing and can
communicate over long distances using infrasound (low-frequency sound waves), they
cannot emit and interpret the reflected sound waves necessary for echolocation.
Relevance: Echolocation is a technique used by bats, dolphins and other animals to determine the location of
objects using reflected sound.

14
Vajiram & Ravi Prelims (2024)
PowerUp All India Mock Test - 01
Q22.
Answer: d
Explanation:
● Bioindicators are living organisms that can be used to assess the health of an ecosystem. They are
sensitive to changes in environmental conditions like pollution, climate change, and habitat loss, making
them valuable tools for environmental monitoring and conservation.
○ Lichens: They are sensitive to atmospheric pollution, which makes them good bioindicators of
air quality. Many epiphytic lichen species have narrow environmental niches and are extremely
sensitive to changes in air pollution. Scientists can track changes in epiphytic lichen community
structure to evaluate changes in the amount of nitrogen and sulphur air pollution in an area. So,
pair 1 is correctly matched.
○ Dragonfly larvae: It is the juvenile form of dragonflies that live underwater and can serve as
bioindicators of mercury. Mercury is a pollutant that can have toxic effects on human and wildlife
health, causing issues with reproductive success or muscle function and more. So, pair 2 is
correctly matched.
○ Plant leaves: Certain plants act as ozone bioindicators and exhibit distinct leaf injuries with
locally elevated ozone concentrations. Ozone forms in the air at ground level from both human-
caused pollution and natural sources. Ozone causes damage to living tissues; in plants, this
damage can cause visible leaf injury, as well as injuries that are harder to observe like reduced
growth or seed production. So, pair 3 is correctly matched.
○ Mosses: They are good indicators of moisture levels and air quality. Their presence or absence
can provide information about microclimates and potential environmental changes. Studies have
shown that mosses can effectively accumulate lead from the air, particularly in areas with heavy
traffic or lead-based industries. High lead levels in moss can serve as an early warning of
potential exposure risks for humans and wildlife. So, pair 4 is correctly matched.
So, all four of the above pairs are correctly matched.
Therefore, option (d) is the correct answer.
Relevance: Recently the experts stated that the bioindicators played a crucial role in the detection of high
pollution levels in the rivers of Maharashtra.

Q23.
Answer: b
Explanation:
● Ocean acidification is mainly caused by carbon dioxide (CO₂) gas in the atmosphere dissolving into the
ocean. It leads to a lowering of the water's pH, making the ocean more acidic.
● When carbon dioxide is released into the atmosphere through human activities such as the burning of
fossil fuels and deforestation, a significant portion of it is absorbed by the world's oceans. CO₂ dissolves
readily in water, forming carbonic acid (H₂CO₃), which dissociates into hydrogen ions (H⁺) and
bicarbonate ions (HCO₃⁻).
● When CO2 is absorbed by seawater, a series of chemical reactions occur resulting in the increased
concentration of hydrogen ions, causing the ocean pH to decrease. CO2 increase causes the seawater to
become more acidic and causes carbonate ions to be relatively less abundant.
○ Carbonate ions are an important building block of structures such as sea shells and coral
skeletons.

15
Vajiram & Ravi Prelims (2024)
PowerUp All India Mock Test - 01
○ Decreases in carbonate ions can make building and maintaining shells and other calcium
carbonate structures difficult for calcifying organisms such as oysters, clams, sea urchins, shallow
water corals, deep sea corals, etc.
Therefore, option (b) is the correct answer.
Relevance: Recently, researchers have expressed concern about the condition of ocean acidification, stating
that by the end of the century, the ocean's pH levels might approach 7.8.

Q24.
Answer: b
Explanation:
Solar engineering techniques are a set of methods and technologies that aim to deliberately alter the climate
system to alleviate the impacts of climate change. These techniques focus on decreasing the amount of sunlight
reaching the Earth’s surface to cool the planet.
● Iron fertilisation involves the addition of iron to ocean waters to stimulate the growth of phytoplankton,
which absorbs carbon dioxide (CO2) during photosynthesis. This technique aims to enhance the ocean's
capacity to absorb CO2 from the atmosphere, thus reducing its concentration and mitigating climate
change. So, point 1 is correct.
● The spreading of silicate rocks on Earth's surface, often referred to as enhanced weathering, involves
the application of finely crushed rocks rich in silicate minerals. It is a carbon capture and storage strategy.
When these rocks weather, they react with atmospheric CO2, effectively sequestering it and reducing
its concentration. So, point 2 is correct.
● Cloud thinning and stratospheric aerosol injections are methods of solar radiation management. They
do not help in removal or the reduction of CO2.
○ Cloud thinning, also known as cloud whitening or cloud brightening, is a solar engineering
technique for solar radiation management. This approach involves increasing the reflectivity of
clouds, typically marine stratocumulus clouds, by introducing small particles or aerosols into
them. The aim is to enhance the clouds' ability to reflect sunlight into space, thereby increasing
their albedo and reducing the amount of solar radiation that reaches the Earth's surface. So,
point 3 is not correct.
○ Stratospheric aerosol injections involve the injection of aerosol particles, such as sulphur
dioxide into the stratosphere. These particles reflect a portion of incoming solar radiation into
space, thus reducing the amount of sunlight reaching the Earth's surface. It is one of the solar-
engineering techniques of solar radiation management. So, point 4 is not correct.
So, only two of the above techniques of carbon dioxide removal from the atmosphere.
Therefore, option (b) is the correct answer.
Relevance: Solar engineering techniques have been gaining traction after the recent ‘Emissions Gap Report’
warned about breaching the warming mark of 2 degrees Celsius over the pre-industrial era.

16
Vajiram & Ravi Prelims (2024)
PowerUp All India Mock Test - 01
Q25.
Answer: c
Explanation:
● The Godavari is the largest of the
peninsular river and is the second
largest basin in India that accounts for
nearly 9.5% of the total geographical
area of the country. It falls in the Deccan
plateau and extends over States of
Maharashtra (48.7%), Andhra Pradesh
and Telangana (23.7%), Chhattisgarh
(12.4%) and Odisha (5.7%) in addition to
smaller parts in Madhya Pradesh
(7.8%), Karnataka (1.4%) and Union
Territory of Puducherry (0.01%). So,
points 1, 3, 4 and 5 are correct.
● Tamil Nadu is not part of the Godavari
basin. So, point 2 is not correct.
So, only four of the above are part of the Godavari basin.
Therefore, option (c) is the correct answer.
Knowledge Box

Important facts about Godavari river:


● Source: It rises at Trimbakeshwar in the Nashik district of Maharashtra, which is located
in Trimbak hills in the Western Ghats.
● Drainage: Godavari river and its principal tributaries drain into the Bay of Bengal by
forming the delta which is gradually extending into the sea and consists of a wide belt
of river borne alluvium.
● Drainage network: It presents an ideal dendritic pattern, a result of uniform lithology
and inadequate structural control.
● Major hills: Its basin is bounded on the north by the Mahadeo Hills, the Satmala Hills,
on the north-west by the Ajanta Range, on the west by the North Sahyadri range of the
Western Ghats, on the east and southeast by the Eastern Ghats and on the south by
the Balaghat Range.
● Major tributaries: Darna, the Pravara and the Manjra are its right bank tributaries,
whereas Kadwa, the Purna, the Kaddam, the Pranhita, the Indravati and the Sabari are
its left bank tributaries.
● Major protected areas: Indravati National Park, Tadoba National Park, Bor Wildlife
Sanctuary, Eturnagaram Wildlife Sanctuary, Manjira Bird Sanctuary.
● Important dams and irrigation projects: The Bhandardara Dam is the highest (82.35
meters height) and Sriram Sagar (SRSP) / Pochampad dam is the longest (15.6 km) dam
in this basin. Wardha sub-basin has the highest number of dams.
● Important pilgrimage sites: Trimbakeshwar temple (Nashik, Maharashtra), Shri Hazur
Sahib Gurudwara (Nanded, Maharashtra), Basara Saraswathi Temple (Adilabad,
Andhra Pradesh), Bhadrachalam Temple (Khammam, Andhra Pradesh), etc.

17
Vajiram & Ravi Prelims (2024)
PowerUp All India Mock Test - 01
Relevance: According to the experts, the major dams / reservoirs on the Godavari river and its tributaries have
been brimming with water due to heavy rain.

Q26.
Answer: c
Explanation:
Alpine vegetation: It is found in the Himalayan ranges between 3,000 metre and 4,000 metre altitude in the
form of dense vegetation cover. It degenerates into a low evergreen shrub on the southern slopes and dry
xerophytic vegetation on the northern slopes of the Himalayas.
● Alpine pastures with stunted conifers below and snowfields above occur at an altitude of about 2250
metre and 2750 metre in the Pir Panjal range. These are used as grazing grounds by the Gujjar nomads
who practice transhumance.
○ Transhumance refers to the seasonal movement of people with their livestock between
geographical or climatic regions.
● In the Western Himalaya, Alpine species with white flowers called '‘Brahmakamal’ and the ‘Kuth’ are
utilized in perfumery. Important tree varieties of alpine vegetation include Silver Fir, Juniper, Pine and
Birch.
● Brahmakamal: It is a species of flowering plant named after Brahma – the Hindu god of creation. It is
considered the ‘king of Himalayan flowers’. It usually blooms in the alpine Himalayan habitats – at the
upper reaches of the mountain ranges between 4500–5500 m. It has also been reported from Myanmar
and South-West China. Brahmakamal is one of those flowers that blooms for only one night; that too,
during late summer or early autumn.
● Kuth: It is an indigenous herb naturally found at an altitudinal range of 2000−3500 m above mean sea
level in subalpine regions of the north western Himalaya and its neighbouring areas.
Therefore, option (c) is the correct answer.
Relevance: Brahmakamal bloom was recently witnessed in many parts of Uttarakhand.

Q27.
Answer: b
Explanation:
● The Gangetic plains exhibit a remarkably homogeneous
surface with little relief. It is a featureless alluvial fertile
plain formed by the rivers' deposits. The plain may be
divided into the following belts:
○ Bhabar belt
○ Terai belt
○ Bhangar belt
○ Khadar belt
○ Delta plains
● Bhabhar: It is about 7-15 kilometers wide narrow belt
adjacent to foothills of Himalayas. It is made up of porous, rocky soils, and pebbles that get deposited
at the foot of the Himalayas due to the breakoff slope. This material is made of debris washed down
from higher ranges. Due to the very high porosity of this belt, water seeps down and the streams are
lost and start flowing underground.

18
Vajiram & Ravi Prelims (2024)
PowerUp All India Mock Test - 01
● Terai: It belt is a 15-30 kilometer wide marshy tract that lies next to Bhabar running parallel to it. Due
to bedrock, the streams lost in Bhabar reappear in Terai. Thus, the water table is very high making the
land marshy. Consequently, Terai belt is moist and thickly forested and is home to a variety of wildlife.
● Bhangar: It is the largest part of the Northern plains made up of old alluvium and forms an alluvial
terrace above flood plains. The soil is made up of old alluvium and in the drier parts, it has calcareous
deposits called kankar.
● Khadar: It is found along all rivers in the flood plains. Soil is made up of new alluvium which gets
deposited every year due to floods.
● Delta Plains: The deltaic plain has a huge amount of new alluvium and is mostly in the form of Khadar.
It covers about 1.9 lakh square kilometers of area in the lower reaches of the Ganga River. It is an area
of deposition as the river flows in this tract sluggishly. The deltaic plain mainly consists of old mud, new
mud and marsh.
So, the correct sequence of the belts of Gangetic plains from North to South direction is Bhabhar-Terai-
Bhangar-Khadar-Delta Plains.
Therefore, option (b) is the correct answer.
Relevance: A new study from the Mahamana Centre of Excellence in Climate Change Research warned that
rainfall patterns over the Indo-Gangetic plains may shift dramatically in the future years as a result of climate
change.

Q28.
Answer: a
Explanation:
● Ansupa Lake is the largest freshwater lake of Odisha situated in Banki sub-division of Cuttack district.
○ The wetland is an oxbow lake formed by River Mahanadi and is spread over an area of 231 ha.
It is also a Ramsar Site. So, point 1 is not correct.
● Tampara Lake is among the most prominent freshwater lakes in Odisha situated in Ganjam district.
○ The depression on the ground gradually filled with rainwater from catchment flow and was
called “Tamp” by the British and subsequently termed “Tampra” by the locals. It is also a Ramsar
Site. So, point 2 is not correct.
● Renuka Lake is the largest natural lake in Himachal Pradesh. This lake is named after the goddess
Renuka. It is fed by a small stream flowing from the lower Himalayan out to the Giri river. It is also a
Ramsar Site. So, point 3 is not correct.
● Thol Lake of Gujarat is a shallow reservoir dominated by open water areas that was originally
constructed for irrigation in 1912. So, point 4 is correct.
○ In 1988, it was declared as a wildlife sanctuary to protect the birdlife found there. It is also a
Ramsar Site.
○ It is on the Central Asian Flyway and more than 320 bird species can be found, making up some
57% of all the bird species of Gujarat.
So, only one of the above lakes is an artificial/man-made lake in India.
Therefore, option (a) is the correct answer.
Knowledge Box

● India has the largest network of Ramsar Sites in Asia. India now has 75 Wetlands of
International Importance designated as Ramsar sites covering an area of 1.33 million
ha across the country.

19
Vajiram & Ravi Prelims (2024)
PowerUp All India Mock Test - 01
Relevance: The National Green Tribunal, Eastern Zone, has directed the Odisha government not to go ahead
with ‘illegal’ construction in and around Tampara Lake.

Q29.
Answer: d
Explanation:
● Lithium, sometimes referred to as “white gold” due to its light colour and high market value, is regarded
as a critical component of the energy transition. It is, at present, a much more precious resource than
fossil fuels. Only 22 million metric tonnes of lithium reserves are presently available worldwide, whereas,
in 2020, the world’s total oil reserves were estimated to be 236.29 billion tonnes. Most of the mineral is
confined to the so-called Lithium Triangle – Bolivia, Argentina and Chile – which has more than 75% of
the world’s lithium reserves. India has been importing lithium for decades.
● The lightest metal in the world, lithium is commonly used in electric vehicles, cellphones and
rechargeable batteries for laptops.
Therefore, option (d) is the correct answer.
Relevance: The Principal Scientific Advisor to the Government of India recently said that India should explore
alternatives to lithium-ion batteries for scaling up electric mobility.

Q30.
Answer: a
Explanation:
● The Tibetan plateau plays an important role in initiating the monsoon circulation over the Indian
subcontinent. The plateau of Tibet is more than 600 kms wide in the west and 1000 kms in the East. Its
length from west to east is about 2000 kms. The average height of the plateau is about 4000 m. Thus, it
is an enormous block of high ground acting as a formidable barrier. It has 2º to 3º higher temperatures
than the neighbouring areas. This acts as a high-level heat source. This 'Heat Engine' produces a thermal
anticyclone over the region. So, statement 1 is correct.
● The anticyclone at Tibet weakens the western subtropical jet streams south of the Himalayas but
produces tropical easterly jet streams on the southern side of the anticyclone. This tropical easterly jet
stream first develops in longitudes east of India and then extends westward across India and the Arabian
Sea to eastern Africa. These upper-level easterly jets create a flow of air on the southern side of the
Tibetan plateau that reaches down to low levels over northern India. So, statement 2 is correct and
statement 3 is not correct.
● During summer, the insolational heating of air over the Tibetan Plateau weakens the western
subtropical jet stream south of the Himalayas with the resultant reversal of pressure gradient and wind
flow over northern India. All these conditions result in the burst of southwest monsoon over the Indian
subcontinent.
Therefore, option (a) is the correct answer.
Relevance: Recently, experts stated that the Nitrogen Aerosols are ‘threatening the fragile ecosystem’ in the
Himalayan & Tibetan Plateau (HTP) region.

20
Vajiram & Ravi Prelims (2024)
PowerUp All India Mock Test - 01
Q31.
Answer: a
Explanation:
● Saline and Alkaline Soils: They are mainly spread across the arid and semi-arid tracts of Rajasthan,
Punjab, Haryana, Uttar Pradesh and Bihar. Such soils have sodium, calcium, and magnesium salt and
encrustations on the surface. These soils are infertile and are also known by different names such as
Reh, Kallar, Usar, Thur, Rakar, Karl and Choppen.
● The origin of such soils is mainly from decomposed rock and mineral fragments which on weathering
liberate sodium, magnesium and calcium salts and sulfurous acid. These salts are transported to rivers
by the process of solution and percolate on the subsoils of the plains.
● In areas of canal irrigation and higher sub-soil water table, these harmful salts are transferred from
below to top-soils through the process of evaporation, which makes the soil unfit for agriculture.
Therefore, option (a) is the correct answer.

Q32.
Answer: a
Explanation:
● Foliar application of fertilization is a technique of feeding plants by applying liquid fertilizer directly to
their leaves. It has been used as a means of giving supplemental doses of major and minor nutrients,
plant hormones, stimulants and other beneficial substances.
● In foliar application, the absorption takes place through the stomata and the epidermis. Transport is
usually faster through the stomata, but total absorption may be as great through the epidermis.
● It is effective for the application of minor nutrients like iron, copper, boron, zinc and manganese.
Further it provides several benefits to plants which include increased yield, better resistance to diseases
and pests, improved drought tolerance and enhanced crop quality.
Therefore, option (a) is the correct answer.
Relevance: The recently announced NAMO Drone Didi scheme will help in the foliar application of liquid
fertilisers like Nano Urea and Nano DAP.

Q33.
Answer: c
Explanation:
● The Vedic texts mentions terms like Sabha, Samiti and Sansad. These terms are significant evidence of
egalitarian perspective and public participation in governance. The Rigveda states that the Samiti should
be constituted fairly and with the principle of inclusivity. The Atharvaveda uses the term, Narishta
(desired by humans), for Sabha. They also refer to participatory institutions like the Sansad. So,
statement 1 is correct.
● The Vinaya Pițaka, the Buddhist rule book that was written after the Buddha’s death, details the
discipline to be followed in the monastic community. It describes an election process, the method of
voting in it; the constitution of Santhagara (an assembly), the election of the Mahasammata (the great
elect), the process of moving a resolution, and what quorum, consent, dissent and referendum means.
So, statement 2 is correct.

21
Vajiram & Ravi Prelims (2024)
PowerUp All India Mock Test - 01
● The Licchavi General Assembly and the council of nine elected members worked together. All the issues
and differences of opinion were settled by a vote of the majority, Yebhuyyasika. The elections were
carried out in Santhagarasala. One person elected in the assembly, designated as Asana-pannapaka, was
assigned to make the seating arrangements. Salakas (chips or wooden sticks) were used as voting paper
and votes were called chhanda. So, statement 3 is not correct.
○ Trial by ordeal refers to an ancient practice wherein the guilt or innocence of people accused of
crimes was decided by forcing them to undergo painful and dangerous ordeals. The accused was
declared to be either innocent or guilty based on whether he or she survived the ordeal without
dying or being severely hurt. It was believed that the innocent undergoing the ordeal would be
saved by god.
● The Ukkal inscriptions at Vishnu Temple of Parantaka Chola I and Rajakesarivarman inform regarding
the direct involvement of people in the governance. They detail the election process for local
governance including the constitution of wards, qualifications and disqualifications of candidates and
voters, and tenure of the elected candidates. So, statement 4 is correct.
Therefore, option (c) is the correct answer.
Knowledge Box

Vinaya Pitaka:
● It also describes three voting procedures that still have resonance: a secret ballot that
employs voting sticks; whispering one’s preference into another monk’s ear and open
voting through the show of hands.
● It put the presiding officer in charge of detecting unfair practices, such as voting twice
or conducting a ballot without ensuring a quorum.
● Disputes regarding interpretation of rules were to be resolved by a committee elected
for the purpose, and by a majority vote in the monastic assembly.

Q34.
Answer: b
Explanation:
● In the late Vedic period, the governance of towns, villages and temples, groups of merchants were
assigned responsibilities throughout India. Puga or the local administration and Nigama or the city
administration, are mentioned in the texts and inscriptions. In these institutions, people were allowed
to express their views freely.
● Narada in his Narada Smriti recommends, “while making laws for the state, the king should study the
laws and customs of castes, social organisations, Puga (town administration), Nigama (city
administration) and Shrenis (merchant guilds) of the state and include them in it."
Therefore, option (b) is the correct answer.

Q35.
Answer: c
Explanation:
● Svetambaras (wearers of white clothes) and Digambaras (the naked) are the two important sects of
Jainism. Their division took place shortly after the death of Mahavira when the community split into
several sects. Their division was on the basis of nudity. The literal meaning of the word Digambara is
sky-clad and that of Svetambara is white-clad. These two sects are divided into a number of sects.

22
Vajiram & Ravi Prelims (2024)
PowerUp All India Mock Test - 01
● The Digambaras contend that perfection cannot be reached by anyone who wears clothing. They think
that a man should abstain from food and possessions, including clothing, to become a saint. The
Digambaras strictly maintain that there can be no salvation without nakedness.
● The Svetambaras believe that having known that the true self consists in the freedom from passions,
having realised the strength of the spiritual practice of non-attachment, and having understood the
gradual order of undertaking the practice of the means of liberation, one can very well understand a
monk’s acceptance of clothing. The only essential point is that when one attains the state of perfect
non-attachment, one definitely attains liberation, irrespective of one’s being nude or not. Clothing is
not an obstacle to salvation. It is the attachment that acts as an obstacle to salvation. So, statement 1
is correct.
● Svetambara recognises the existence of canonical texts and 12 Angas. According to Svetambara
tradition, the agamas were composed in the first Jain council (300 BC) at Patliputra. The canons of
Svetambara are composed of twelve Angas, twelve Upangas, ten Prakirnakas, four Mulasutras, six
Chedasutras, and two Chulika sutras. So, statement 2 is correct.
● Only the Svetambaras decorate the temple idol with clothing and ornaments. The Digambara authors
put more emphasis on mental culture than on idol worship. So, statement 3 is not correct.
● The Svetambaras allow women to enter the monastic order under the assumption that they can attain
Nirvana. They believe tirthankaras can be men or women and say that Mallinatha (19th Tirthankara)
began her life as a princess. As per the Digambara belief, since women cannot go without clothes, they
are said to be incapable of salvation. So, statement 4 is correct.
So, only three of the above statements are correct.
Therefore, option (c) is the correct answer.
Relevance: Jain community in the United States has recently announced a nationwide digital detox movement.

Q36.
Answer: c
Explanation:
● Greek historians Megasthenes and Dionysius resided at the Mauryan court of Chandragupta Maurya
(321BC - 297BC). Megasthenes in his excerpt from Indica explains the advisory body of the king as
follows: “The seventh class consists of the council lords of the state, who advise the king, or the
magistrates of self-governed cities, in the management of public affairs.”So, statement 1 is correct.
● The Greek historian, Diodorus Siculu writes that at the time of Alexander’s invasion (in 326 B.C.), most
cities in North West India had democratic forms of government (though some areas were under kings,
e.g. Ambhi and Porus) and this is also mentioned by the historian Arian. He observed that laws in India
ordained that no one among them shall, under any circumstance be a slave. All will possess equal rights
and democratic governments were set up in cities. So, statement 2 is correct.
● Fa-Hien in his travelogue reflected on the rule of law, public welfare and elements of equality. He
described that kings had been firm believers in law. They made their offerings to a community of monks,
took off their royal caps and along with their relatives and ministers, supplied them with food with their
own hands. They would sit down in front of the chairman on the carpet with the community itself. So,
statement 3 is correct.
So, all three of the above statements are correct.
Therefore, option (c) is the correct answer.
Relevance: Recently, the ‘Bharat: Mother of Democracy’ portal was launched at the G20 Summit that has
reference to the foreigners’ accounts of ancient India.

23
Vajiram & Ravi Prelims (2024)
PowerUp All India Mock Test - 01
Q37.
Answer: d
Explanation:
The Vijayanagara Empire was established by the sons of Sangama, Harihara and Bukka in 1336. It had its capital
on the banks of river Tungabhadra.
● A distinct feature of the Vijayanagara state was the importance of the Brahmans as political and secular
personnel rather than ritual leaders. Most of the Durga dannaiks (in charge of forts) were Brahmans.
Literary sources substantiate the theory that fortresses were significant during this period and were
placed under the control of the Brahmans, especially of Telugu origins. So, statement 1 is correct.
○ The Brahmans also played an important role as military commanders in the Vijayanagara army.
For example, under Krishna Deva Raya, Brahman Timma received economic support as he was
an integral part of the political system.
○ Differentiation was made between crown villages and amaram villages (whose income was
under the charge of the local military chiefs).
● There are references to three major categories of land tenure: Amara, bhandaravada and manya. These
indicate how the village income was distributed. The bhandarvada was a crown village comprising the
smallest category. A part of its income was utilised to maintain the Vijaynagara forts. Income from the
manya (tax-free) villages was used to maintain the Brahmans, temples, and mathas. The largest category
was of the Amara villages given by the Vijaynagar rulers to the amaranayakas. Their holders did not
possess proprietary rights in the land but enjoyed privileges over its income only. The Amara tenure
was primarily residual in the sense that its income was distributed after deductions had been made for
the support of the Brahmans and forts. Three-quarters of all the villages came under this category. So,
statement 2 is correct.
● Land revenue was the major source of the state's income. Another category of land rights through
which income was derived was a result of investment in irrigation. It was called dasavanda in Tamil
country; and Kattu-Kodage in Andhra and Karnataka. This kind of agrarian activity concerning irrigation
was undertaken in semi-dry areas where hydrographic and topographic features were conducive for
carrying out developmental projects. The dasavanda or Kattu-Kodage was a share in the increased
productivity of the land earned by the person who undertook such developmental work (e.g.
construction of a tank or channel). This right to income was personal and transferable. So, statement 3
is correct.
Therefore, option (d) is the correct answer.
Knowledge Box

● Besides land tax, many professional taxes were also imposed.


○ These were shopkeepers, farm servants, shepherds, washermen, potters,
shoemakers, musicians etc.
● There was also property tax.
● Grazing and house taxes were also imposed.
● Villagers were also supposed to pay for the maintenance of the village officers.
● Sthala dayam, margadayam and manula dayam were three major transit dues.

24
Vajiram & Ravi Prelims (2024)
PowerUp All India Mock Test - 01
Q38.
Answer: b
Explanation:
● The Bakhshali Manuscript is an ancient Indian mathematical text written on birch bark and is considered
to be the earliest recorded example of the use of zero.
● The seminal text, dating back roughly to the third or fourth century A.D., is in one of the Bodleian
Libraries of the University of Oxford.
● The first documented use of zero came from the ancient astronomer and mathematician Brahmagupta.
Brahma Gupta's text Brahmasphutasiddhanta, written in 628 A.D., is the first text to talk of zero as a
number in its own right and to include a discussion of the arithmetic of zero, including the act of dividing
by zero.
Therefore, option (b) is the correct answer.
Knowledge Box

National Manuscripts Authority (NMA):


● It is an autonomous body under the Union Ministry of Culture, which is mandated to
preserve the vast manuscript wealth of India. The NMA would have the powers of a
civil court to regulate the allocation of access to manuscripts and would also have an
investigation wing for the purpose of conducting an inquiry into thefts and desecration
of texts. It would also ensure that the manuscripts are not lost by damage or theft.
● According to the NMA, 75% of the existing manuscripts are in Sanskrit and 25% are in
regional languages. Manuscripts are written on materials such as palm leaf, paper, cloth
and bark on themes ranging from history and religion to literature, astrology and
agricultural practice.
Relevance: The Government of India is planning a law on the protection of Indian manuscripts.l

Q39.
Answer: c
Explanation
Mamallapuram (Mahabalipuram):
● It is located along the Coromandel Coast and was a port city of the Pallavas. The town’s religious centre
was founded by a 7th-century-CE Hindu Pallava king, Narasimhavarman, also known as Mamalla. So,
pair 1 is correctly matched.
● Ancient Chinese, Persian and Roman coins found at Mamallapuram point to its earlier existence as a
seaport. It contains many surviving 7th-and 8th-century Pallava temples and monuments, chief of which
are the sculptured rock relief popularly known as “Arjuna’s Penance,” or “Descent of the Ganges,” a
series of sculptured cave temples, and a Shaiva temple on the seashore.
● The town’s five rathas, or monolithic temples, are the remnants of seven temples, for which the town
was known as Seven Pagodas.
Muziris:
● In ancient times, coastal south India functioned as a hub between the Roman Empire in the west and the
south-east Asian countries in the East.
● Many important trading centres developed, the most famous of which was Muziris. It has been referred
to in the ancient Tamil text, the Purananuru as a bustling port city under the Chera rulers. So, pair 2 is
correctly matched.
Puhar (Poompuhar and Kaveripattinam):
25
Vajiram & Ravi Prelims (2024)
PowerUp All India Mock Test - 01
● It was an important port during the reign of the Chola kings, Rajaraja I and Kulothunga. They established
good relationships with China and trade between the two countries flourished. The articles exported
during this period were cotton fabrics, spices, drugs etc. So, pair 3 is correctly matched.
Tamralipti:
● According to literary and archaeological evidence, the port town of Tamralipti flourished from 3rd
century BCE to 8th century CE. Tamralipti was the most prominent port of the Kalinga kingdom.
● Historians speculate that it was the desire to access the rich ports on the Eastern coast like Tamralipti
that Ashoka invaded Kalinga. Tamralipti became an important port in the Mauryan empire. So, pair 4
is not correctly matched.
● It also played a very important role in spreading Buddhism to South and South East Asia. King Asoka
sent his son Mahendra and daughter Sanghamitra to transport the Bodhi Tree to Sri Lanka, and introduce
Buddhism to Sri Lanka through the Tamralipti port.
So, only three of the above pairs are correctly matched.
Therefore, option (c) is the correct answer.

Q40.
Answer: b
Explanation:
● The National Calendar of India is based on the Saka Calendar which has been adopted as the official
civil calendar besides the Gregorian Calendar. The Saka era is believed to have been founded by King
Shalivahana of the Satavahana dynasty in 78 AD. The Saka era marks the commemoration of King
Shalivahana’s major military victories.
● The Saka Calendar is recognized beyond Indian borders. It is used by the Hindus of prominent South East
Asian countries like Java, Bali and Indonesia. Bali celebrated Nyepi, translated as the Day of Silence, on
March 22 or Saka New Year. Nepal Samvat is clearly an evolution of the Saka Calendar.
Therefore, option (b) is the correct answer.
Knowledge Box

● The Saka Calendar is based on luni-solar reckoning of time. The calendar consists of 365
days and 12 months like the normal Gregorian calendar. Chaitra is the first month of
the year beginning on March 22. During leap years, the starting day of Chaitra
corresponds with March 21. The names of the months in Saka Calendar are:
○ Chaitra (March 21 – April 20)
○ Vaishakha (April 21-May 21)
○ Jyeshtha (May 22-June 21)
○ Ashadha (June 22- July 22)
○ Shravana (July 23-August 22)
○ Bhadra (August 22-September 22)
○ Ashwin (September 23-October 22)
○ Kartika (October 23-November 21)
○ Agrahayana (November 22-December 21)
○ Pausha (December 22-January 20)
○ Magha (January 21- February 19) and
○ Phalguna (February 20-March 20/21)

26
Vajiram & Ravi Prelims (2024)
PowerUp All India Mock Test - 01
● The Saka calendar's most noted use is in the Gazette of India, alongside the commonly
used Gregorian calendar. All India Radio Broadcasts follow the Saka Calendar. Other
official Government of India Calendars, timetables, documents and communications
refer to the Saka Calendar dates.

Q41.
Answer: b
Explanation:
● The Persian term for land revenue during the Mughal rule was mal and mal wajib. Kharaj was not in
regular use, as it was under Delhi Sultanate. The process of land revenue collection had two stages:
assessment (tashkhis/ jama) and actual collection (hasil). Assessment was made to fix the state
demand. On the basis of this demand, actual collection was done separately for kharif and rabi crops.
○ Under the Delhi Sultanate, the Islamic land tax with which the new rulers of India were familiar
was kharaj. The kharaj was essentially a share in the produce of the land and not a rent on the
land.
● Under the Mughals, assessment was separately made for kharif and rabi crops. After the assessment
was over, a written document called patta, qaul or qaul-qarar was issued in which the amount or the
rate of the revenue demand was mentioned. The assessee was in return supposed to give qabuliyat i.e.
‘the “acceptance” of the obligation imposed upon him, stating when and how he would make the
payments’. So, statement 1 is correct.
● In Mughal India, the Zabti system was the most important method of assessment. The origin of this
practice is traced to Sher Shah. During Akbar’s reign, the system was revised a number of times before
it took the final shape. So, statement 2 is not correct.
○ The main features of the Zabti system as it finally came into operation under Akbar were:
■ measurement of land was essential
■ fixed cash revenue rates known as dastur-ul amal or dastur for each crop
■ all the collection was made in cash
● Ghalla Bakhshi (crop-sharing) was a land revenue system under the Mughals. In some areas it was called
bhaoli and batai. The Ain-i Akbari notes three types of crop-sharing:
○ Division of crop at the threshing floor after the grain was obtained. This was done in the presence
of both the parties in accordance with agreement.
○ Khet batai: The share was decided when the crop was still standing in the fields, and a division
of the field was marked.
○ Lang batai: The crop was cut and stacked in heaps without separating grain and a division of crop
in this form was made.
● In Malikzada’s Nigarnama-i Munshi (late 17th century) crop sharing has been mentioned as the best
method of revenue assessment and collection. Under this method, the peasants and the state shared
the risks of the seasons equally. Under Ghalla Bakhshi, the state’s share was collected directly from the
field. In other systems, the state collected its share at the time of harvest. So, statement 3 is correct.
So, only two of the above statements are correct.
Therefore, option (b) is the correct answer.

27
Vajiram & Ravi Prelims (2024)
PowerUp All India Mock Test - 01
Q42.
Answer: a
Explanation:
● Regulating Act of 1773: It was the first step taken by the British Government to control and regulate the
affairs of the East India Company in India. It recognized for the first time the political and administrative
functions of the Company. The Act fixed the tenure of the Governor-General at five years, renewable
with the approval of the Court of Directors of the East India Company. So, statement 1 is correct.
● Act of Settlement, 1781: This Act was passed by the British Parliament in a bid to rectify the defects of
the Regulating Act of 1773. It was also known as the Amending Act of 1781. It exempted the Governor-
General and the Council from the jurisdiction of the Supreme Court for the acts done by them in their
official capacity. Similarly, it also exempted the servants of the company from the jurisdiction of the
Supreme Court for their official actions. So, statement 2 is not correct.
● Charter Act, 1793: It gave the Governor-General more powers and control over the governments of the
subordinate Presidencies of Bombay and Madras. It provided that the Commander-in-Chief was not to
be a member of the Governor-General’s council unless he was so appointed. So, statement 3 is not
correct.
So, only one of the above statements is correct.
Therefore, option (a) is the correct answer.

Q43.
Answer: d
Explanation:
● Sattriya dance in modern form was introduced by the Vaishnava Saint Sankaradeva in the 15th century
AD in Assam. The art form derives its name from the Vaishnava monasteries known as ‘Sattras’, where
it was primarily practised. It finds mention in the ancient text ‘Natya Shastra’ of sage Bharat Muni. It is
inspired by the Bhakti Movement.
● Some of the features of Sattriya dance include:
○ It was an amalgamation of various dance forms prevalent in Assam, mainly Ojapali and Devdasi.
○ The focus of the Sattriya recitals is to own the devotional aspect of dance and narrate the
mythological stories of Vishnu.
○ It also includes Nritta, Nritya and Natya.
○ The dance is generally performed in groups by male monks known as ‘Bhokots’ as part of their
daily rituals or even at festivals.
○ Khol (drum), Cymbals (Manjira) and Flute form the major accompanying instruments of this
dance form.
○ The songs are compositions of Sankaradeva known as ‘Borgeets’.
○ It combines both Lasya and Tandava elements.
○ In the modern times, Sattriya dance has evolved into two separate streams – the Gayan-Bhayanar
Nach and the Kharmanar Nach.
○ Ankia Naat, a type of Sattriya, involves play or musical drama. It was originally written in an
Assamese-Maithili mixed language called Brajavali. Another similar form is ‘Bhaona’, which is
based on stories of Lord Krishna.
Therefore, option (d) is the correct answer.

28
Vajiram & Ravi Prelims (2024)
PowerUp All India Mock Test - 01
Knowledge Box

● Chhau: It is a form of mask dance that uses vigourous martial movements to narrate
mythological stories. Some narrations also use natural themes such as Sarpa nritya
(serpent dance) or Mayur nritya (peacock dance).
● Bihu: It is the popular dance of Assam, performed in groups by both men and women.
The dancers are dressed in colourful traditional dresses to celebrate the pomp and
gaiety. The dance performance includes group formations, rapid hand movements and
brisk footsteps.
● Cheraw: It is a folk dance of Mizoram and is performed using bamboo sticks. It is likely
to have a foreign origin. Men tap long pairs of bamboo in rhythmic beats and girls dance
to the beats of bamboo.

Q44.
Answer: c
Explanation:
● Chirala and Perala, two tiny villages in Andhra Pradesh's Prakasam district (then in Madras Province's
Guntur district), have carved out a prominent place in the history of Indian independence. The two
villages had a combined population of 15,000, who paid a tax of Rs 4,000. In 1919, the then-Madras
government combined the twin villages into a municipality and raised the tax to Rs. 40,000.
● A majority of the residents who were already paying a hefty tax fiercely opposed this and started a ‘no-
tax’ campaign under the leadership of Duggirala Gopalakrishnayya, a young graduate from Edinburg
University. So, statement 1 is correct.
● On the advice of Mahatma Gandhi, Duggirala Gopalakrishnayya decided to move the people out of the
village ‘en masse’, which would then result in the municipal legislation being abolished.
○ In 1921, Rama Dandu', a volunteer corps with discipline and a red uniform, was formed that
relocated the people. As a result, approximately 13,000 people fled their villages. So, statement
2 is correct.
● However, the satyagraha failed due to their dwindling resources, Duggirala's imprisonment in
Tiruchirappalli and the absence of leaders to carry on the struggle in his absence.
● Nevertheless, despite the campaign's failure, the tenacity and courage these villagers displayed proved
useful to them in the later stages of the independence movement. Additionally, this satyagraha planted
the seeds for the Non-Cooperation Movement in India. Gopalakrishnayya was given the title of 'Andhra
Rathna.'
Therefore, option (c) is the correct answer.

Q45.
Answer: c
Explanation:
● The Earth rotates anti-clockwise when viewed from the North Pole. This rotation, from west to east, is
why one sees the Sun rising in the east and setting in the west. The surface velocity of rotation varies
from point to point on the Earth. It is about 1600 km per hour or about 460 metres in a second near the
equator.

29
Vajiram & Ravi Prelims (2024)
PowerUp All India Mock Test - 01
● The velocity gradually reduces as one moves to the poles and it is practically zero there. A satellite
launched from the sites near the equator towards the east direction will get an initial boost equal to the
velocity of Earth's surface. This is similar to an athlete circling round and round before throwing a discus
or a shot put.
● Artificial satellites are launched from the Earth in the eastward direction because the rotational speed
of the Earth is highest at the equator due to its rotation on its axis from west to east, and launching in
the eastward direction takes advantage of this rotational speed, providing additional velocity to the
satellite. So, statement 1 is correct.
● The rotational motion of the Earth is from west to east and this rotation imparts an initial eastward
velocity to objects on the Earth's surface. So, statement 2 is not correct.
So, Statement–I is correct but Statement–II is incorrect.
Therefore, option (c) is the correct answer.
Relevance: India’s Chandrayaan-3 mission was launched from the Satish Dhawan Space Center (SDSC) in
Sriharikota — an island on the east coast of Andhra Pradesh.

30
Vajiram & Ravi Prelims (2024)
PowerUp All India Mock Test - 01
Q46.
Answer: d
Explanation:
● The far side of the Moon is the side that is not visible from Earth but has its own day and night cycle. It
is often referred to as “dark side of the Moon”. This does not mean “dark” as in the absence of light, but
rather “dark” as in the unknown. In reality, both the near and far sides receive (on average) almost equal
amounts of light directly from the Sun.
● The Moon and Earth are "tidally locked" so the moon rotates on its axis at the same speed it orbits
Earth. This is one always sees the same side of the moon. The unseen side, known as the far side, is
shielded from radio interference at night by the moon's mass. While the far side of the Moon offers
the radio silence needed for studying the Dark Ages, it's also a harsh environment where scientific
equipment struggles to survive.
● The US Department of Energy (DoE) and NASA scientists are collaborating to land a radio telescope on
the far side of the Moon. This effort might lead to humanity's first exploration of the Dark Ages of the
universe.
○ The near side of the Moon provides a static place for Earth-observing arrays.
Therefore, option (d) is the correct answer.
Knowledge Box

Tidal Locking:
It is the phenomenon by which a body has the same rotational period as its orbital period
around a partner. This indicates that the body rotates once around its axis for each orbit it
makes around a particular body in space.
Relevance: Recently, India's Chandrayaan-3 probe is trying out its landing optics by taking pictures of the moon's
far side.

Q47.
Answer: a
Explanation:
● Recently, researchers have developed BonFIRE or bond-selective fluorescence-detected infrared-
excited spectro-microscopy. It is a cutting-edge microscopy technique that combines fluorescence and
vibrational microscopy.
● BonFIRE technique offers unparalleled single-molecule imaging and uses isotopes to create various
vibrational colors, providing deep insights into biological molecules and processes.
● BonFIRE combines two microscopy techniques into one process with greater selectivity and sensitivity,
enabling researchers to visualize biological processes at the unprecedented single-molecule level and
understand biological mechanisms from a molecular point of view.
Therefore, option (a) is the correct answer.

31
Vajiram & Ravi Prelims (2024)
PowerUp All India Mock Test - 01
Knowledge Box
Techniques Behind BonFIRE:
● Fluorescence microscopy- It images molecules and other microscopic
structures by tagging them with fluorescent chemical markers, causing them
to glow when imaged.
● Vibrational microscopy- It makes use of natural vibrations in the bonds that
hold together the atoms of a molecule. A sample to be imaged is bombarded
with light, in this case infrared light. That bombardment causes the bonds of
the material’s molecules to vibrate in such a way that their type can be
identified.
Relevance: Recently, Scientists have developed a new microscopy technique called BonFIRE.

Q48.
Answer: c
Explanation:
Large Language Model (LLM):
● It is a type of Artificial Intelligence (AI) algorithm that uses deep learning techniques and massively
large data sets to understand, summarise, generate and predict new content.
● They use transformer models and are trained using massive datasets. This enables them to recognize,
translate, predict, or generate text or other content. LLMs are based on Neural Networks (NNs), which
are computing systems inspired by the human brain. There are multiple large language models
developed. Examples include the GPT-3 and GPT-4 from OpenAI, LLaMA from Meta and PaLM2 from
Google. These are models that can understand language and can generate text.
● Application of LLMs include Translation, Malware Analysis, Content Creation, Virtual Assistants,
Chatbots, Customer Support, Detecting and Preventing Cyber Attacks, Code Development,
Transcription, Sales Automation and Sentiment Analysis of texts. So, points 1, 2, 4 and 5 are correct.
● LLMs currently are not capable of self-correcting their reasoning in the absence of external help. Certain
researchers have shown that large language models struggle to intrinsically self-correct reasoning
mistakes without external guidance. So, point 3 is not correct.
Therefore, option (c) is the correct answer.
Relevance: The Principal Economic Advisor has recently said that India will explore building Large Language
Models.

Q49.
Answer: d
Explanation:
● Saturn is the sixth and the second largest planet in the Solar System and is known for its bright rings.
Saturn’s axis of rotation has a 26.7 degree tilt. Its ring system is also tilted to the plane of Saturn’s orbit.
As a result, when Saturn revolves around the Sun, it seems to nod up and down when viewed from Earth
and the view of its rings also keeps changing.
○ Apart from Saturn, Jupiter, Uranus and Neptune have their own ring systems.
● Saturn takes 29.5 years to complete an orbit around the Sun and every 13 to 15 years, the edge of its
rings aligns directly with Earth. As the rings are very thin in most places, they reflect very little light, and
are very difficult to see, making them essentially invisible. Saturn’s rings will not be visible from Earth
because they will be perfectly aligned with the line of sight. The rings will gradually return to view by
2032 as the planet will continue to revolve around the Sun.
32
Vajiram & Ravi Prelims (2024)
PowerUp All India Mock Test - 01
Therefore, option (d) is the correct answer.
Relevance: The National Aeronautics and Space Administration (NASA) has predicted that Saturn’s rings will
disappear by 2025.

Q50.
Answer: b
Explanation:
Quantum dots:
● They are man-made nanoscale crystals
that exhibit unique optical and
electronic properties, including the
ability to transport electrons and emit
light of various colours when exposed
to ultraviolet light. QDs can absorb
light of a shorter wavelength and are
more capable of absorbing ultraviolet
light than in absorbing blue light. So,
statement 1 is not correct.
○ When semiconductor particles are made small enough, they exhibit quantum effects, which
restrict the energies at which electrons and holes (the absence of electrons) can exist within the
particle.
○ As energy is linked to wavelength (or colour), it can be used for tuning the optical properties of
the particle based on its size. By controlling the size of the particle, it can be made to emit or
absorb specific wavelengths (colours) of light.
○ Quantum dots can also be used to harness solar radiations. They can absorb the incoming
sunlight and create excitons (electron-hole pairs).
● Larger quantum dots emit lower energy (longer wavelength) light. The largest quantum dots in the set
will emit red-orange light and the smallest quantum dots will emit aqua blue light (high energy). So,
statement 2 is correct.
● Certain cancer treatments use quantum dots for targeted drug delivery and other therapeutic
measures. This has wider applications in the field of nanomedicine too. Quantum dots can be used as
security markers on currency and documents as an anti-counterfeit measure. Broadly, they can be used
as fluorescent markers to tag and track objects. So, statement 3 is correct.
Therefore, option (b) is the correct answer.
Knowledge Box

● They are particularly significant for optical applications owing to their bright, pure
colours along with their ability to emit a rainbow of colours coupled with their high
efficiencies, longer lifetimes and high extinction coefficient. Examples include Light
Emitting Diodes (LEDs) and solid state lighting and displays. In photovoltaic cells, they
improve the absorption and efficiency in converting solar light into electricity.
● Examples of applications taking advantage of these unique electronic properties include
transistors, solar cells, ultrafast all-optical switches and logic gates, and quantum
computing, among many others.

33
Vajiram & Ravi Prelims (2024)
PowerUp All India Mock Test - 01
● The small size of dots allow them to go anywhere in the body making them suitable for
different biomedical applications like medical imaging, biosensors, etc. Nanoscale-
sized quantum dots are also used to map biological tissues by biochemists.
Relevance: The 2023 Nobel Prize in Chemistry has been awarded for the discovery and synthesis of quantum
dots.

Q51.
Answer: a
Explanation:
● Article 244 in Part X of the Constitution of India envisages a special system of administration for certain
areas designated as ‘Scheduled Areas’ and ‘Tribal Areas’.
● In the Constitution of India, the expression “Scheduled Areas” means such areas as the President may
by order declare to be Scheduled Areas.
● The President is empowered to declare an area to be a scheduled area. He can also increase or decrease
its area, alter its boundary lines, rescind such designation or make fresh orders for such re designation
on an area in consultation with the Governor of the State concerned.
Therefore, option (a) is the correct answer.
Relevance: Recently, the Supreme Court has held that Parliamentary or State law wouldn’t apply to Scheduled
Areas only if the Governor notifies so.

Q52.
Answer: c
Explanation:
The Directive Principles of State Policy are enumerated in Part IV of the Constitution of India from Articles 36
to 51. They denote the ideals that the State should keep in mind while formulating policies and enacting laws.
● According to Article 51 of the Constitution of India, the State shall endeavour to:
○ Promote international peace and security
○ Maintain just and honourable relations between nations
○ Foster respect for international law and treaty obligations in the dealings of organised peoples
with one another
○ Encourage settlement of international disputes by arbitration. So, points 1, 3 and 4 are correct.
● Article 51A in Part IVA of the Constitution of India encompasses the Fundamental Duties to be fulfilled
by every citizen of India. Some of them are:
○ To uphold and protect the sovereignty, unity and integrity of India. So, point 2 is not correct.
○ To value and preserve the rich heritage of the country’s composite culture
○ To develop a scientific temper, humanism and spirit of inquiry and reform
○ To safeguard public property and to abjure violence.
So, only three of the above are the Directive Principles of State Policy enumerated in Part IV of the
Constitution of India.
Therefore, option (c) is the correct answer.

34
Vajiram & Ravi Prelims (2024)
PowerUp All India Mock Test - 01
Q53.
Answer: c
Explanation:
● An Electoral Bond is a bearer instrument in the nature of a Promissory Note and an interest-free
banking instrument which can be encashed by an eligible political party only through a designated bank
account with the authorised bank.
○ An Electoral Trust is a trust or a legal arrangement in which a body holds property for the sake
of the political party. ETs are set up by companies with the sole objective of distributing the
contributions received by them from other companies and individuals to the political parties. So,
statement 1 is correct.
● Electoral Trusts provide transparency in funding political parties by revealing contributors and
beneficiaries, allowing the public to be informed about contributors and beneficiaries of a particular
trust. In contrast, Electoral Bonds, exempt from disclosure requirements, do not disclose the names of
contributors and beneficiaries, limiting transparency. So, statement 2 is correct.
● The Income-tax (First Amendment) Rules, 2013, states that the Electoral Trusts shall accept
contributions only by way of an account payee cheque drawn on a bank or account payee bank draft or
by electronic transfer to its bank account and shall not accept any contribution in cash. Also, the
purchaser of Electoral Bonds is allowed to buy these bonds only on due fulfilment of all the extant KYC
norms and by making payment from a bank account.
Therefore, option (c) is the correct answer.
Relevance: The Supreme Court concluded hearing the petitions challenging the Electoral Bond Scheme, 2018
and has reserved its verdict.

Q54.
Answer: d
Explanation:
Under Article 356 of the Constitution of India, the President's rule can be imposed on any State of India on the
grounds of the failure of the constitutional machinery. It is also called a State Emergency or Constitutional
Emergency.
● The President acquires extraordinary powers when the President’s rule is imposed in a State where he
can take up the functions of the State Government and powers vested in the Governor or any other
executive authority in the State.
● When the President’s rule is imposed in a State, the President dismisses the State Council of Ministers
headed by the Chief Minister. Then the State Governor, on behalf of the President, carries on the State
administration with the help of the Chief Secretary of the State or the advisors appointed by the
President.
● When the State Legislature is suspended or dissolved, the Parliament can delegate the power to make
laws for the State to the President or to any other authority specified by him in this regard.
● When the Parliament is not in session, the President can promulgate ordinances with respect to the
State's administration.
Therefore, option (d) is the answer.
Relevance: The Supreme Court recently held that the declaration of State emergency under Article 356 and
subsequent actions of the President should have a reasonable nexus.

35
Vajiram & Ravi Prelims (2024)
PowerUp All India Mock Test - 01
Q55.
Answer: b
Explanation:
● Democracy is considered one of the best forms of government because it ensures liberty of thought,
expression, belief, faith and worship, equality of status and opportunity, fraternity as well as the right to
participate in political decision-making.
● The words, “democratic” and “decentralization” form the key to the understanding of the expression
“democratic decentralization”. The word “democratic” explains the nature and purpose of the concept
as well as its basic claims in an institutional set-up. The word ‘decentralization’ is essentially indicative
of the method to realize the end as contained in the word ‘democratic’. Decentralization means the
transfer of planning, decision-making or administrative authority from the Central Government to its
field organizations, local administrative units, semi-autonomous organizations, local governments or
non-governmental organizations.
● There exist a difference between democratic decentralization and delegation. Delegation means the
grant of authority from a superior to a subordinate, to be enjoyed not as a right but as a derived
concession and that also to be exercised at the pleasure of the superior. The term ‘democratic
decentralization’ on the other hand means grant of authority by a superior to a subordinate as a right
to be enjoyed by the subordinate and not as a concession.
● India has an indirect form of democracy where laws and regulations are not framed directly by the
people but indirectly by the representatives elected by the people.
Therefore, option (b) is the correct answer.
Knowledge Box

● Federalism: It is a system of government in which the power is divided between a


central authority and various constituent units of the country. Its basic principle is that
the legislative and executive authority is partitioned between the Centre and the States
not by any law to be made by the Centre but by the Constitution itself.
● Delegated legislation: It is a common practice where Parliament entrusts certain
functions to authorities established by law. This delegation is necessary because
lawmakers cannot address every detail directly. The specific powers granted to these
authorities are documented in statutes.

Q56.
Answer: b
Explanation:
● Central Consumer Protection Authority (CCPA): It is a statutory body under the Consumer Protection
Act, 2019, Ministry of Consumer Affairs, Food and Public Distribution. Its objective is to regulate
matters relating to violation of consumer rights and to promote, protect and enforce the rights of
consumers as a class. So, statement 1 is correct.
● CCPA acts as an investigation wing headed by a Director General (appointed by the Central Government)
whereas the Central Consumer Protection Council (CCPC) is an advisory body on consumer issues,
headed by the Union Minister of Consumer Affairs, Food and Public Distribution with the Minister of
State as Vice Chairperson and 34 other members from different fields.

36
Vajiram & Ravi Prelims (2024)
PowerUp All India Mock Test - 01
● CCPA can conduct investigations into violation of consumer rights and institute complaints/prosecution
either suo motu or on a complaint received or on the directions from the Central Government and
recommend appropriate remedial measures for their effective implementation. So, statement 2 is
correct.
○ It can file complaints or intervene in any proceedings before the district commission or the state
commission or the national commission in respect of any allegation of violation of consumer
rights or unfair trade practices.
● A person aggrieved by any order passed by the CCPA may file an appeal to the National Commission
within a period of thirty days from the date of receipt of such order. So, statement 3 is not correct.
○ The National Consumer Disputes Redressal Commission (NCDRC), India is a quasi-judicial
commission in India which was set up in 1988 under the Consumer Protection Act of 1986.
So, only two of the above statements are correct.
Therefore, option (b) is the correct answer.
Relevance: Recently, the Central Consumer Protection Authority (CCPA) has imposed penalties on coaching
institutes for making misleading claims in their advertisements.

Q57.
Answer: c
Explanation:
● The office of ‘whip’ is mentioned neither in the Constitution of India nor in the Rules of the House nor
in a Parliamentary Statute. It is based on the conventions of the Parliamentary government.
● The Chief Whip of the government party in the Lok Sabha is the Minister of Parliamentary Affairs. In
the Rajya Sabha, the Minister of State for Parliamentary Affairs holds this position.
● The Chief Whip is directly responsible to the Leader of the House. It is part of his duties to advise the
Government on parliamentary business and to maintain a close liaison with the Ministers in regard to
parliamentary business affecting their departments.
● The Chief Whip is assisted by one or two Deputy Ministers and Ministers of State.
● Functions: The main function of the Whips is to keep members of their party within sound of the division
bell whenever any important business is under consideration in the House. Whips are responsible for
the attendance of the members at the time of important divisions.
○ Whips, both of the ruling party and those of the opposition, play a very significant role in the
smooth and efficient functioning of parliamentary democracy.
○ The Chief Whip is the eyes and ears of the leader of the party so far as the members are
concerned.
○ He conveys the wishes of the leader to the members of the party and keeps the leader informed
of the current opinion in the party as also the moods and inclinations of individual members when
these deserve special notice.
Therefore, option (c) is the correct answer.
Relevance: The Telangana government recently appointed four legislators as government whips the name of
the chief whip, however, is yet to be announced.

37
Vajiram & Ravi Prelims (2024)
PowerUp All India Mock Test - 01
Q58.
Answer: c
Explanation:
Diplomatic immunity: It is a form of legal protection and policy held between two nations to safeguard
diplomats and ensure their safe passage when on foreign soil. Those who represent their country abroad enjoy
diplomatic immunity.
● The Vienna Convention on Diplomatic Relations of 1961 and the Vienna Convention on Consular
Relations of 1963 codified most diplomatic and consular practices, including diplomatic immunity.
● The acts of diplomacy may be performed by the Head of State, Government, Minister of Foreign
Relations, or Diplomatic Agents. Diplomatic immunity is applicable to all of them.
● Premises of a Foreign Diplomatic Mission, Members of the family of a Diplomatic Agent, Members of the
administrative and technical personnel of a Mission are provided diplomatic immunity by the
Government of India. So, points 1, 2 and 3 are correct.
● Locally recruited personnel of Foreign Diplomatic Missions do not enjoy any immunity from civil,
administrative or criminal jurisdiction in India. So, point 4 is not correct.
So, only three of the above are provided with diplomatic immunity by the Government of India.
Therefore, option (c) is the correct answer.
Relevance: Recently, Canada removed 41 diplomats after India threatened to revoke their diplomatic immunity.

Q59.
Answer: a
Explanation:
Northeast India is famous for its cultural heritage and diverse social groups. These groups have their own
traditional administrative councils to take decisions on local issues.
● Dzumsa: It is a general council i.e. a traditional system of self-governance in two villages namely
Lamteng and Lachung in Chungthang administrative division of the North District of Sikkim. It is
authorised by the State Government to exercise the powers and functions provided under the Sikkim
Panchayat Act.
● Jamatia Hoda: It is the highest council in the traditional social system established by the Jamatia clan
more than 400 years ago in Tripura. The Jamatia clan had a three-tier social structure consisting of Hoda
(apex body), Moyal (at regional level) and Luku (at village level) for smooth interaction and social order.
● Kebang: It is a traditional village council, a form of local self-government, of the Adi tribes of Arunachal
Pradesh that looks after the administration of justice in the Adi-inhabited areas. It also acts as indigenous
traditional court system looking after the administration of justice in the Adi inhabited areas as an
instrument of Alternative Dispute Resolution.
Therefore, option (a) is the correct answer.
Relevance: Recently, the former ‘Okra’ (chief) of ‘Hoda’ triggered a controversy by suggesting that converting
tribals should be dropped from the Scheduled Tribe (ST) category.

Q60.
Answer: a
Explanation:
● The Death Penalty in India is awarded in exceptional cases that fall under the “rarest of rare” doctrine
and not life imprisonment. So, statement 1 is not correct.
○ The Supreme Court (SC) propounded the 'rarest of rare' doctrine in the landmark Bachan Singh
v/s State of Punjab case in 1980.

38
Vajiram & Ravi Prelims (2024)
PowerUp All India Mock Test - 01
○ The SC declared that life imprisonment is the rule in serious cases and the death penalty is an
exception.
● In the landmark case of Bhagirath & Ors v. Delhi Administration (2012), SC declared that a convict
undergoing life imprisonment is expected to remain in custody till the end of his natural life, subject
to any remission granted by the appropriate Government.
o The SC said, “it appears to us there is a misconception that a prisoner serving a life sentence has
an indefeasible right to be released on completion of either fourteen years or twenty years
imprisonment. The prisoner has no such right”. So, statement 2 is not correct.
● Section 31 of the Criminal Procedure Code (CrPC), 1973 says that sentences shall run consecutively
unless the court says otherwise when a person is convicted for more than one offence in the same trial.
● In 2014, a bench of SC had ruled that in cases where life imprisonment has been awarded, all other
sentences will have to run concurrently. So, statement 3 is correct.
o The court’s logic was that there can be no sentence worse than that which consumes the full
span of a man’s life.
o A Constitution Bench has also clarified that two or more life sentences have to run concurrently
and not consecutively.
So, only one of the above statements is correct.
Therefore, option (a) is the correct answer.
Knowledge Box

Evolution of Life Imprisonment in India:


● In India, the British brought the concept of transportation of prisoners as a reform to
death penalty or mutilation which was largely in practice during the ancient and
medieval period.
● The prisoners of the English East India Company were transported to Andaman Island.
● In 1955, Section 53 of Indian Penal Code, 1860 was amended and imprisonment for life
was added substituting transportation as a punishment.
● Thus, in India, life imprisonment was introduced in 1955 and it came into effect in
1956.
Relevance: A Special Bench of the Supreme Court was formed to hear a petition challenging the release of 11
men sentenced to life imprisonment in a criminal case.

Q61.
Answer: a
Explanation:
● Mumbai Interbank Offer Rate (MIBOR) is the rate at which banks borrow unsecured funds from one
another in the interbank market. At present, it is used as a reference rate for floating rate notes,
corporate debentures, term deposits, interest rate swaps and forward rate agreements. The pricing of
overnight indexed swaps, a type of overnight interest rate swap used for hedging interest rate risk is
based on overnight MIBOR.
● MIBOR is calculated based on input from a panel of 30 banks and primary dealers. MIBOR was first
established in 1998 and modeled after the London Interbank Offer Rate (LIBOR).
Therefore, option (a) is the correct answer.

39
Vajiram & Ravi Prelims (2024)
PowerUp All India Mock Test - 01
Knowledge Box

● Marginal Standing Facility (MSF) rate is the interest rate at which the Reserve Bank of
India (RBI) provides money to the scheduled commercial banks that are facing an acute
shortage of liquidity.
● London Interbank Offer Rate (LIBOR) is the global reference rate for unsecured short-
term borrowing in the interbank market. It acts as a benchmark for short-term interest
rates. It is used for pricing of interest rate swaps, currency rate swaps as well as
mortgages.
● Repo rate is the rate at which RBI lends money to commercial banks in the event of any
shortfall of funds. It is used to control inflation.

Q62.
Answer: c
Explanation:
CIBIL Report is provided by the Credit Information Bureau (India) Limited (CIBIL), a Credit Information
Company (CIC). Its primary function is to collect and maintain financial data as provided by the lenders to further
generate and provide credit reports and credit scores for its customers.
● A CIBIL report contains detailed information on the credit already taken (like home loan, car loan,
overdraft facilities, credit card, personal loan, etc). It also mentions the repayment history of the money
borrowed over a period of time. The following information is provided in a CIBIL credit report:
○ CIBIL Score: It is a three-digit numeric summary of the credit history. It takes into account
borrowers’ credit profiles over the last 36 months. The value of Credit Score may range between
300 to 900. CIBIL scores are also known as the CIBIL Transunion score as CIBIL India is part of
TransUnion, an American multinational group. Credit sanctioning is carried out according to the
CIBIL scores.
○ Personal Information: It includes name, gender, date of birth and identification numbers, like
PAN, passport number and voter’s number.
○ Contact Information: Information about the address and telephone numbers.
○ Employment Information: It includes monthly or annual income details, as reported by the
members (banks and financial institutions). So, points 3 and 4 are correct.
○ Account Information: It consists of the details of credit facilities like name of the lending
institutions, kind of credit facilities (personal, home, overdraft, etc.), account numbers, date
opened, date of last payment, loan amount, ownership details, current balance and a month-on-
month record of payments.
○ Enquiry Information: It includes the details of enquiries listed every time when a person applies
for a loan or credit card. These enquiries get listed in the report and will contain the vendor’s
name, type of credit, amount of credit applied and date of application. So, point 1 is correct.
● A CIBIL Report does not contain details of savings, investments or fixed deposits. So, point 2 is not
correct.
So, only three of the above details are included in the Credit Information Bureau (India) Limited (CIBIL) Report.
Therefore, option (c) is the correct answer.

40
Vajiram & Ravi Prelims (2024)
PowerUp All India Mock Test - 01
Knowledge Box

● Credit Information Companies:


○ They collect public data, credit transactions, and payment histories of
individuals and companies regarding loans and credit cards, among others.
○ Their primary function is to gather data from various sources, such as banks,
financial institutions, lenders, and other credit-granting entities, and then
compile this data into credit reports.
○ CICs in India are licensed by the RBI and governed by the Credit Information
Companies Regulation Act, 2005 (CICRA) and various other rules and regulations
issued by the RBI.
○ At present, four credit information companies have been given certificates of
registration by the RBI. These companies are Credit Information Bureau (India)
Limited (CIBIL), Equifax Credit Information Services Private Limited, Experian
Credit Information Company of India Private Limited and CRIF High Mark Credit
Information Services Private Limited.
Relevance: Recently, the Kerala High Court stayed an order that said that a student’s credit score cannot be a
factor in rejecting an education loan application.

Q63.
Answer: d
Explanation:
● Appreciation of currency is the rise of the value of domestic currency in contrast to other foreign
currencies. It occurs when the exchange rate for a currency rises over time.
● Currency appreciation happens when the demand for a currency increases or if its supply decreases in
the share market. As a currency appreciates, more units of the second currency (foreign) can be bought
with one unit of the first currency (home currency). For example, if the Indian rupee appreciates against
the US dollar, INR will buy more dollars than before. This means that when the currency of that country
becomes stronger, global investors are more likely to invest in the stock market of that country. Thus,
foreign investors buy assets denominated in the particular asset of the country.
● Fluctuations in currency (appreciation or depreciation) has a huge impact on trade and capital flows in
the economy, thus influencing the level of money supply, inflation and rate of growth in the economy.
● Appreciation of currency benefits importers as they have to pay less in domestic currency for imported
goods. For example: for the domestic economy, appreciation makes foreign travel and education abroad
cheaper. Since the cost of imports declines, it leads to a decline in the rate of inflation in the domestic
economy. However, it has a negative impact on exporters as their goods become more expensive for
foreign buyers which can also lower the demand.
Therefore, option (d) is the correct answer.

Q64.
Answer: a
Explanation:
● Monetary base is also known as Reserve Money (M0), High-Powered Money, base money etc.
○ M0 = Currency in Circulation + Bankers’ Deposits with RBI + Other deposits with RBI. So, points 1
and 3 are correct.

41
Vajiram & Ravi Prelims (2024)
PowerUp All India Mock Test - 01
● M1 = Currency with public + Demand deposits with the Banking system (current account, saving account)
+ Other deposits with RBI
● M2 = M1 + Savings deposits of Post Office Savings Banks. So, point 2 is not correct.
● Broad Money (M3) = M1 + Time deposits with the banking system. So, point 4 is not correct.
● M4 = M3 + All deposits with post office savings banks
Therefore, option (a) is the correct answer.

Q65.
Answer: c
Explanation:
Essential Services Maintenance Act (ESMA), 1968:
● It was enacted to maintain certain essential services and the normal life of the community. The Act
includes a long list of “essential services" in its charter ranging from post and telegraph to railway,
hospitals, airport and port operations.
● Each State has its own ESMA, with provisions that differ slightly from the federal statute. As a result, if
the nature of the strike disturbs only one or more States, the States can initiate it. The Act also allows
States to choose the essential services to enforce ESMA.
● The Government is empowered to prohibit employee strikes if it feels that the strike is gravely
disturbing public life. The Government has to issue a general or special order to end the strike. Any strike
becomes illegal after the passing of this order. Also, the employees cannot cite bandhs or a curfew as an
excuse not to report to work. If their work is considered necessary for the maintenance of any of the
essential services, the employees can't refuse to work overtime. So, statements 1 and 2 are correct.
● It gives police the right to arrest anyone violating the Act’s provisions without a warrant. Once the Act
has been invoked by the government, any person who commences a strike or otherwise takes part in
any such strike shall be punishable with imprisonment for a term which may extend to six months, or
with a fine which may extend to two hundred rupees, or with both. It also makes the instigation of
strikes punishable by law. So, statement 3 is correct.
● Any establishment dealing with the production, supply or distribution of petroleum, coal, power, steel,
fertilizers etc. also falls under the essential services category. So, statement 4 is not correct.
So, only three of the above statements are correct.
Therefore, option (c) is the correct answer.
Relevance: The Odisha Government recently invoked the Orissa Essential Services (Maintenance) Act, 1988
prohibiting strikes by paramedical staff.

42
Vajiram & Ravi Prelims (2024)
PowerUp All India Mock Test - 01
Q66.
Answer: b
Explanation:
● Convertibility is the ease with which a country's currency can be converted into gold or another currency
through global exchanges.
● India has full Current Account Convertibility, but it does not have full Capital Account Convertibility.
● The Current Account includes the
transaction of goods, services, primary
income and secondary income between
the residents and the rest of the world.
● Thus, the Indian Rupee is fully convertible
for all the transactions related to the
export and import of goods and services
and remittances. So, points 1 and 4 are
correct.
● The Capital Account records all
international transactions of assets. An
asset is any one of the forms in which
wealth can be held, for example: money,
stocks, bonds, Government debt, etc.
● The Capital Account is the summation of
Foreign Direct Investment (FDI), Foreign
Portfolio Investment (FPI), other
investments (trade credit, loans and
deposits) and reserve accounts.
● Thus, the Indian Rupee is only partially
convertible for transactions related to
External Commercial Borrowing, FDI and
FPI. So, points 2 and 3 are not correct.
So, the Indian Rupee is fully convertible in only
two of the above transactions.
Therefore, option (b) is the correct answer.
Relevance: The European Union has suggested that India liberalise its capital account restrictions as part of a
comprehensive free trade agreement being discussed with it.

Q67.
Answer: d
Explanation:
● Asian Premium is an extra charge being collected by the Organization of the Petroleum Exporting
Countries (OPEC) countries from Asian countries when selling oil in comparison to Western countries.
The discriminatory Asian Premium is mainly used by OPEC countries to subsidise Western buyers at the
cost of Asian buyers.
● The Asian Premium has its origins in the late 1980s when Saudi Arabia, the de-facto OPEC leader,
adopted a marker-based price system for its oil exports – West Texas Intermediate (WTI) for the US,
Brent for Europe and Dubai/Oman for Asia. The Asian market was costlier than the American and
European ones and did not fully reflect market dynamics.

43
Vajiram & Ravi Prelims (2024)
PowerUp All India Mock Test - 01
o This was due to the lack of substantial production centres (other than in the Gulf region) and the
oil derivatives market in Asia. Also, Asian countries, dependent heavily on oil imports, were
essentially price-takers. In short, the oil exporters used their superior bargaining power to
squeeze Asian oil importers while charging less to their American and European customers.
Therefore, option (d) is the correct answer.
Knowledge Box

Organization of the Petroleum Exporting Countries (OPEC)


● It is a group of the world’s major oil-exporting nations. It was founded in 1960 to
coordinate the petroleum policies of its members and to provide member states with
technical and economic aid.
● It is a cartel that aims to manage the supply of oil to set the price of oil on the world
market, to avoid fluctuations that might affect the economies of both producing and
purchasing countries.
● Member countries: Iran, Iraq, Kuwait, Saudi Arabia and Venezuela (the five founders),
plus Algeria, Angola, Congo, Equatorial Guinea, Gabon, Libya, Nigeria and the United
Arab Emirates.
Relevance: As per the analysts, western sanctions on Russian and Iranian oil have channelled cheap fuel to Asia.

Q68.
Answer: b
Explanation:
● Priority Sector Lending (PSL) is a policy of the Reserve Bank of India (RBI) that requires banks to lend a
certain percentage of their net credit to specified sectors of the economy that otherwise find it difficult
to obtain credit.
● This may include agriculture, Micro, Small and Medium Enterprises (MSME), education, housing, social
infrastructure, renewable energy and others.
● Under the category of ‘others’, the RBI has included the following sub-categories:
○ Loans to Farmer Producer Organisations (FPOs) engaged in agriculture and allied activities, with
a credit limit of up to ₹2 crore per borrowing entity. So, point 1 is correct.
○ Bank loans up to ₹30 crore to borrowers for purposes like setting up Compressed Biogas plants
(CBG), with a cap of ₹2 crore per plant. So, point 2 is correct.
○ Loans to start-ups are provided, as per the definition of the Ministry of Commerce and Industry,
that are engaged in activities other than agriculture or MSME, up to ₹50 crore per
borrower/unit. So, point 3 is correct.
● Following sectors/entities are excluded from the provisioning of PSL in India:
○ Electric vehicles (EVs) are not explicitly mentioned as sub-categories under PSL. Therefore, they
are not eligible for the PSL. The Government in consultation with the Reserve Bank of India (RBI)
is considering a proposal to include EVs in the PSL category. So, point 4 is not correct.
Therefore, option (b) is the correct answer.
Relevance: The Government in consultation with the Reserve Bank of India (RBI) is considering a proposal to
include Electric Vehicles (EVs) in the Priority Sector Lending.

Q69.
Answer: b
Explanation:
44
Vajiram & Ravi Prelims (2024)
PowerUp All India Mock Test - 01
● The concept of water footprints refers to the measurement of water consumption associated with
various activities or products, considering the volume of water used directly or indirectly throughout
their production or supply chain.
● Different types of water footprints help understand and quantify water usage in different contexts:
○ Grey Water: It is wastewater from non-toilet plumbing systems such as hand basins, washing
machines, showers and baths. When handled properly, greywater can be safely reused for the
garden. So, pair 1 is not correctly matched.
■ Grey water footprint is the amount of fresh water required to assimilate pollutants to
meet specific water quality standards. The grey water footprint considers point-source
pollution discharged to a freshwater resource directly through a pipe or indirectly through
runoff or leaching from the soil, impervious surfaces, or other diffuse sources.
○ Blue Water: It is the amount of surface or groundwater that is consumed by human activities.
This includes water that is evaporated, incorporated into a product, taken from one body of
water and returned to another at different times. They are an indicator of how much freshwater
is consumed to produce goods and services. So, pair 2 is correctly matched.
○ Black Water: It is a waste that contains urine, faeces, toilet paper, and water from flush toilets.
It also contains organic matter from dishwater drains. Black water requires a septic tank or sewer
system to transport it to a treatment facility to remove pathogens. So, pair 3 is not correctly
matched.
○ Virtual Water: It is the water used in the production of a product or service. It includes the water
used to grow the raw materials and the water used during production. Virtual water is often
"hidden" from the end user. It is involved in the production and trade of food and non-food
commodities and services. Some examples of virtual water include: textiles, machinery,
livestock, crops, smartphones, jeans, electricity etc. So, pair 4 is correctly matched.

So, only two of the above pairs are correctly matched.


Therefore, option (b) is the correct answer.
Relevance: The Stockholm World Water Week of 2023 showcased its longstanding work on water issues through
a series of side events.

Q70.
Answer: a
Explanation:
● The Assessment Report on Invasive Alien Species and their Control (known as the “Invasive Alien Species
Report”) is an assessment report on invasive alien species and their control. It is released by the
Intergovernmental Platform on Biodiversity and Ecosystem Services (IPBES). The United Nations
Environment Programme (UNEP) provides secretariat services to IPBES.
● The Invasive Alien Species Assessment:
○ assesses the array of invasive alien species that affect biodiversity and ecosystem services

45
Vajiram & Ravi Prelims (2024)
PowerUp All India Mock Test - 01
○ analyzes the extent of the threat posed by such species to various categories of biodiversity and
ecosystem services
○ identifies the major pathways for and drivers of the introduction and spread of such species
between and within countries
○ highlights the global status of and trends in the impacts of invasive alien species
○ assesses the effectiveness of current international, national and associated policy options that
could be employed to prevent, eradicate and control invasive alien species
● According to the 2023 report, more than 37,000 alien species have been introduced by human activities
to regions and biomes around the world and the global economic cost of invasive alien species exceeded
$423 billion annually in 2019.
○ Invasive alien species are one of the five major direct drivers of biodiversity loss globally,
alongside land and sea-use change, direct exploitation of organisms, climate change and
pollution.
Therefore, option (a) is the correct answer.
Knowledge Box
International instruments on Invasive Species:
● IPBES is an independent intergovernmental body established by States to
strengthen the science-policy interface for biodiversity and ecosystem services for
the conservation and sustainable use of biodiversity, long-term human well-being
and sustainable development.
○ It was established in 2012 by 94 Governments. It is not a United Nations
body.
● The Kunming-Montreal Global Biodiversity Framework is an international
agreement that aims to halt and reverse the loss of biodiversity. It was agreed upon
at the 15th meeting of the Conference of Parties to the UN Convention on Biological
Diversity.
● The Species Survival Commission (SSC) is a science-based network of conservation
experts that provides technical advice to the International Union for Conservation
of Nature (IUCN).
● The Global Invasive Species Programme (GISP) is an international partnership that
focuses on addressing the global threat of invasive species through policy
development, awareness raising and information exchange.
Relevance: Invasive Alien Species Report was released by the Intergovernmental Platform on Biodiversity and
Ecosystem Services (IPBES) in 2023.

Q71.
Answer: a
Explanation:
● The double-humped camel, also known as the Bactrian camel (Camelus bactrianus), is a native of the
Gobi desert and is found on a vast expanse of cold-desert areas across Mongolia, China, Kazakhstan,
Turkmenistan, Uzbekistan and parts of Afghanistan. So, statement 1 is not correct.
● In India, the double-humped camel only exists in a small population located in the Nubra Valley of
Ladakh. This camel is of immense interest to researchers as it can withstand temperatures as low as
minus-40 degrees Celsius; has high disease resistance; and very high feed conversion efficiency. So,
statement 2 is not correct.
○ Kharai camels are found exclusively in the Kutch district of Gujarat, India.

46
Vajiram & Ravi Prelims (2024)
PowerUp All India Mock Test - 01
● The hair of the double-humped camel is used in the manufacture of high-quality wool fibres viz. shawls,
pullovers, sweaters, winter garments, coats and caps. The high staple length and quality of wool fiber
indicated its usefulness in fiber processing for the village cottage industry. So, statement 3 is correct.
So, only one of the above statements is correct.
Therefore, option (a) is the correct answer.
Knowledge Box

Other facts related to double-humped camel:


● Camels store fat in their humps which can be used for energy when food and water are
scarce. The hump changes size depending on the food availability. During the late
winter when pastures are scanty, the humps collapse. Also, the male animals are
heavier than female.
● The double-humped camel has a greater water-retaining capacity compared to the
single-humped camel (Dromedary). This allows them to survive in harsher desert
environments for longer periods without access to water. Their humps store fat, which
can be metabolized into water when needed.
● The International Union for Conservation of Nature (IUCN) has listed the Wild Bactrian
Camel as a critically endangered species in the Red List of Endangered Species since
2002, owing to multiple threats. In contrast, the domestic Bactrian camel, which is
widely distributed, is not considered to be a threatened species.
Relevance: The Indian Army has deployed double-humped camels for logistical support in eastern Ladakh.

Q72.
Answer: a
Explanation:
Climate Club:
● It is an initiative aimed at cooperation between countries in decarbonising the industrial sector. It is led
by Germany and Chile and was launched at the 28th Conference of Parties (COP28) to the United
Nations Framework Convention on Climate Change (UNFCCC). The Club is claimed to currently
represent more than half of the global economy.
● It aims to decarbonise industries through ambitious policies, alignment of methodologies and standards
and improving finance and assistance for emerging and developed economies. The Club’s core idea is
based on the fact that developed countries account for a significant share of the emissions arising
through steel, cement, and chemicals.
● The programmatic activities of the Club are divided into three key pillars:-
○ Pillar 1: Advancing ambitious and transparent climate change mitigation policies
○ Pillar 2: Transforming industries
○ Pillar 3: Boosting international cooperation and partnership
Therefore, option (a) is the correct answer.
Relevance: Climate Club was recently launched at the 28th UNFCCC - Conference of Parties (COP28) to tackle
industrial emissions.

Q73.
Answer: d
Explanation:
Compressed Biogas Blending Obligation (CBO):
47
Vajiram & Ravi Prelims (2024)
PowerUp All India Mock Test - 01
● It aims to promote the production and consumption of Compressed Biogas (CBG) in India. It is a major
step towards enhancing the use and adoption of CBG. The National Biofuels Coordination Committee
(NBCC) announced the introduction of phase-wise mandatory blending of CBG in Compressed Natural
Gas (CNG) (Transport) and Piped Natural Gas (PNG) (Domestic) segments of the City Gas Distribution
(CGD) sector.
● The key objectives of the CBO are to:
○ stimulate demand for Compressed Biogas (CBG) in the City Gas Distribution (CGD) sector.
○ import substitution for Liquefied Natural Gas (LNG). So, statement 1 is not correct.
○ saving in Forex
○ promoting circular economy
○ assist in achieving the target of net zero emission etc.
● CBO will be voluntary till FY 2024-2025 and mandatory blending obligation will start from FY 2025-26.
CBO shall be kept as 1%, 3% and 4% of total CNG/PNG consumption for FY 2025-26, 2026- 27 and 2027-
28 respectively. From 2028-29 onwards CBO will be 5%. So, statement 2 is not correct.
● A Central Repository Body (CRB) shall monitor and implement the blending mandate based on the
operational guidelines approved by the Union Minister of Petroleum and Natural Gas. So, statement 3
is not correct.
So, none of the above statements are correct.
Therefore, option (d) is the correct answer.
Relevance: The National Biofuels Coordination Committee (NBCC) has recently announced the introduction of
phase-wise mandatory blending of Compressed Biogas.

Q74.
Answer: c
Explanation:
Stubble burning is a common practice of disposing of crop residue by farmers in some parts of India. It is one of
the major sources of air pollution in Delhi and nearby regions, especially during the winter season. Stubble
burning releases various pollutants into the atmosphere, such as particulate matter, carbon monoxide, nitrogen
oxides, sulphur dioxide and volatile organic compounds.
Following pollutants are released into the atmosphere by stubble burning:
● Carbon monoxide (CO): A toxic gas that reduces the oxygen-carrying capacity of blood and can cause
headaches, dizziness, nausea, and even death. Stubble burning emits about 9 million tons of CO annually.
So, point 1 is correct.
● Sulphur dioxide (SO2): A gas that irritates the eyes, nose, throat, and lungs, and can cause acid rain and
respiratory diseases. Stubble burning emits about 0.25 million tons of SO2 annually. So, point 2 is correct.
● Chlorofluorocarbon (CFC): It is a synthetic gas that is used in refrigeration, aerosols, and other
applications. CFCs have no significant natural sources and are mostly man-made. It is not released during
stubble burning. So, point 3 is not correct.
● Polycyclic aromatic hydrocarbons (PAHs): These are a group of carcinogenic compounds that can cause
mutations, tumours, and cancers in humans and animals. Stubble burning emits about 0.004 million tons
of PAHs annually. So, point 4 is correct.
● Methane (CH4): This is another greenhouse gas that is more potent than CO2 in trapping heat and
accelerating global warming. Stubble burning emits about 0.12 million tons of CH 4 annually. So, point 5
is correct.

48
Vajiram & Ravi Prelims (2024)
PowerUp All India Mock Test - 01
● Particulate matter (PM2.5): These are tiny particles that can penetrate deep into the lungs and cause
various health issues, such as coughing, wheezing, shortness of breath, heart attacks, strokes, and
cancer. Stubble burning emits about 1.28 million tons of PM2.5 annually. So, point 6 is correct.
So, only five of the above pollutants are released by stubble burning.
Therefore, option (c) is the correct answer.
Relevance: According to a new study, Delhi had the highest Particulate Matter (PM) 2.5 level in October 2023
for the past five years.

Q75.
Answer: b
Explanation:
● Shyama Prasad Mukherji Tunnel: Also known as Chenani-Nashri Tunnel, it is a road tunnel in Jammu
and Kashmir. It provides a safe, all-weather route to commuters traveling from Jammu Udhampur to
Ramban, Banihal and Srinagar. So, pair 1 is correctly matched.
● Atal Tunnel: It has officially been certified by World Book of Records as the ‘World’s Longest Highway
Tunnel above 10,000 Feet’. It runs under the ‘Rohtang Pass’ in Himachal Pradesh and was constructed
on the Manali - Leh Highway. So, pair 2 is correctly matched.
● Banihal Qazigund Road Tunnel: The tunnel would reduce the road distance between Banihal and
Qazigund by 16 km. It is a twin-tube tunnel – one for each direction of travel – with the twin tubes being
interconnected by a cross passage every 500m for maintenance and emergency evacuation. The tunnel
would help establish an all weather connection between Jammu and Kashmir, and bring the two regions
closer. So, pair 3 is not correctly matched.
● Sela tunnel: It is an under construction tunnel in Arunachal Pradesh. Once completed it will offer the
world the longest bi-lane tunnel at an altitude above 13,000 feet. It will provide all-weather
thoroughfares for the people of Tawang of Arunachal Pradesh. So, pair 4 is not correctly matched.
So, only two of the above pairs are correctly matched.
Therefore, option (b) is the correct answer.
Relevance: Recently, almost 90 per cent of the construction works of Sela Tunnel in Arunachal Pradesh have
been completed.

Q76.
Answer: b
Explanation:
● Basel city in Switzerland lies along the Rhine River, at the mouths of the Birs and Wiese rivers, where
the French, German and Swiss borders meet. It is at the entrance to the Swiss Rhineland. It is the site of
the Bank for International Settlements. So, point 1 is correct.
● Geneva city in Switzerland is situated in the southwestern corner of Switzerland. It is located at the
southwestern end of Lake Geneva at its junction with the Rhone River. Geneva hosts the headquarters
of different international agencies like the International Labour Organisation and the World Trade
Organisation. So, point 2 is not correct.
● Bonn city in Germany is located on the Rhine river. From 1949 to 1990 it was the provisional capital of
West Germany, and it served as the seat of the German federal government from 1990 until 1999–2000.
So, point 3 is correct.
● Berlin is the capital and chief urban centre of Germany. It lies in the wide glacial valley of the Spree
River, which runs through the centre of the city. Berlin is Germany’s largest industrial town and a major
centre of trade and technological development. So, point 4 is not correct.

49
Vajiram & Ravi Prelims (2024)
PowerUp All India Mock Test - 01
So, only two of the above cities are located on the banks of River Rhine.
Therefore, option (b) is the correct answer.
Knowledge Box

● Some other important cities located on the banks of the Rhine River are
Strasbourg/Kehl, Carlsruhe, Mannheim/Ludwigshafen, Mainz, Cologne, Duesseldorf,
Duisburg and Rotterdam.
Relevance: Recently, parts of the river Rhine in south Germany were closed to shipping after heavy rains.

Q77.
Answer: b
Explanation:
The universal force of attraction among all the entities or matter in this universe is also known as gravity (g). It
is the fundamental force responsible for bringing all matter closer. The measurement of gravity is based on the
acceleration it imparts to freely falling objects. On Earth's surface, the acceleration of gravity is approximately
9.8 m/s². This means that for every second an object falls freely, its speed increases by about 9.8 meters per
second. In essence, gravity governs the motion of objects and is the reason why things naturally fall toward the
Earth's center. The reading of the gravity at different places in affected by the following factors:
● The Earth is not perfectly spherical but is an oblate spheroid. The polar radius (radius near poles) of the
Earth is 21 km smaller than its equatorial radius (near the equator). The acceleration due to gravity is
inversely proportional to the square of the radius of the Earth. Therefore, the acceleration due to
gravity increases when one moves from the equator to the pole. So, statement 1 is correct.
● The gravitational force decreases with an increase in altitude. Therefore, places at higher elevations
experience slightly weaker gravity compared to those at lower elevations. However, this is not the
reason for the increase in acceleration due to gravity when one moves from the equator to the poles.
So, statement 2 is correct.
So, both Statement-I and Statement-II are correct but Statement-II is not the correct explanation for
Statement-I.
Therefore, option (b) is the correct answer.
Knowledge Box

● The Earth's rotation creates a centrifugal force that slightly reduces the gravitational
force at the equator compared to the poles.
● The value of gravity decreases if one moves towards the center of the Earth. Within
the Earth's interior, the value of g, which represents the acceleration due to gravity,
gradually decreases compared to its value at the Earth's surface. As one moves deeper
below the Earth's surface, the strength of gravity diminishes. As we approach the
Earth's center, the value of g continues to decrease, eventually reaching zero at the
Earth's core.

Q78.
Answer: b
Explanation:

50
Vajiram & Ravi Prelims (2024)
PowerUp All India Mock Test - 01
● Israel, a country in the Middle East, is located at the eastern
end of the Mediterranean Sea. It is bounded to the north by
Lebanon, to the northeast by Syria, to the east and southeast
by Jordan, to the southwest by Egypt, and to the west by the
Mediterranean Sea. So, points 1, 3 and 4 are correct.
● Israel also shares borders with the Palestinian Territories of
the West Bank and Gaza Strip.
● Turkey and Saudi Arabia do not share borders with Israel. So,
points 2 and 5 are not correct.
So, only three of the above-mentioned countries share a land border
with Israel.
Therefore, option (b) is the correct answer.
Relevance: Experts have opined that the Israel-Hamas war might
escalate into a regional crisis with Iran- backed militias targeting US
troops and assets.

Q79.
Answer: c
Explanation:
● The Earth’s atmosphere is divided into five different layers depending upon the temperature condition.
They are: troposphere, stratosphere, mesosphere, thermosphere and exosphere.
● The thermosphere lies between the exosphere and the mesosphere. “Thermo” means heat, and the
temperature in this layer can reach up to 4,500 degrees Fahrenheit.
○ However, the thermosphere feels very cold because there aren’t enough gas molecules to
transfer the heat to us.
○ This also means there
aren’t enough molecules
for sound waves to travel
through.
● The thermosphere is home to the
International Space Station as it
orbits Earth. This is also where
one will find low Earth orbit
satellites.
Therefore, option (c) is the correct
answer.
Relevance: Recent studies suggest that
the solar storms have heated up the
thermosphere which have put the
satellites there in danger.

Q80.
Answer: a
Explanation:

51
Vajiram & Ravi Prelims (2024)
PowerUp All India Mock Test - 01
● Al Hodeidah: It is Yemen’s fourth largest city and one of the it’s chief ports, situated on the Red Sea. It
has been at the centre of Yemen’s civil war as a result of clashes between Yemeni government forces
and the Houthis. So, pair 1 is correctly matched.
● Golan Heights: It is a plateau in south-western Syria that has a political and strategic significance. In
1967, Israel took control of this plateau from Syria during the Six-Day War, causing most Syrian Arab
residents to flee the region. An armistice line was established and the region came under Israeli military
control. So, pair 2 is not correctly matched.
● Darfur region: It lies in the western part of Sudan, near the borders with Libya, Chad, and Central African
Republic. There is a new surge in ethnically-driven killings in West Darfur as the paramilitary Rapid
Support Forces (RSF) took over the main army base in the state capital, El Geneina. So, pair 3 is not
correctly matched.
● Libreville: It is the largest city and capital of Gabon, a country situated on the western side of the African
continent. It is one of the richest countries of Africa in terms of per-capita income. It recently witnessed
a failed coup attempt by the Republican Guard, a unit of the armed forces. So, pair 4 is not correctly
matched.
So, only one of the above pairs is correctly matched.
Therefore, option (a) is the correct answer.
Relevance: Recently, Gabon witnessed a coup attempt by its armed forces.

Q81.
Answer: a
Explanation:
When one travels from Prayagraj to Amritsar by road, following geographical features/sotes can be observed:
● Yamuna River: This is a major river of northern India that
originates from the Yamunotri Glacier in the Himalayas
and flows through Uttarakhand, Himachal Pradesh,
Haryana, Delhi and Uttar Pradesh. It merges with the
Ganges River at Sangam or Prayagraj, which is a sacred
place for Hindus. So, point 1 is correct.
● Karewas: They are elevated tablelands of lacustrine
deposits found in the Kashmir Valley and the Bhadarwah
Valley of Jammu and Kashmir. They are not present in the
route from Prayagraj to Amritsar, which passes through
the States of Uttar Pradesh, Haryana and Punjab. So,
point 2 is not correct.
● Rajaji National Park: It is also a tiger reserve and is located
in the Shiwaliks, near the Himalayan foothills. It includes
three Uttarakhand districts: Haridwar, Dehradun and Pauri Garhwal. While travelling from Prayagraj to
Amritsar by road, one need not cross the State of Uttarakhand. So, point 3 is not correct.
● Thar Desert: It is also known as the Great Indian Desert. It is located partly in Rajasthan State,
northwestern India, and partly in Punjab and Sindh provinces, eastern Pakistan. Marusthali is a dune-
covered portion of the Thar desert. It is not part of the route from Prayagraj to Amritsar, which lies to
the east of the desert. So, point 4 is not correct.
So, only one of the above geographical features is most likely to be observed when travelling from Prayagraj
to Amritsar by road.
Therefore, option (a) is the correct answer.

52
Vajiram & Ravi Prelims (2024)
PowerUp All India Mock Test - 01
Relevance: The Central Government has recently constituted a committee of experts to examine the
embankments of the Yamuna River.

Q82.
Answer: b
Explanation:
● India possesses 247 islands scattered in the Bay of Bengal (204 islands) and the Arabian Sea (43 islands).
Both groups of islands differ from each other in their geological structure. The Bay of Bengal islands are
associated with Tertiary orogeny while those of the Arabian Sea have a coral origin. So, statement 1 is
not correct.
● In regard to the surface salinity in both the areas, it is seen that the Bay of Bengal waters are less saline
than the waters of the Arabian Sea. The average values are 30-33~oo for the Bay of Bengal and 34-
37Vo~ for the Arabian Sea. These differences are generally explained by the fact that there is
considerable influx of freshwater in the Bay of Bengal owing to the number of large river systems both
on the east coast of India and the Burmese coast leading to the lowering of salinity of the waters of the
Bay.
○ While the absence of any such large river system on the west coast of India combined with the
intrusion of waters of very high salinity from the northern Arabian Sea, the Persian Gulf and the
Red Sea contribute to the high salinity of the Arabian Sea waters. So, statement 2 is correct.
Therefore, option (b) is the correct answer.
Knowledge Box

Arabian Sea Islands:


● The offshore islands situated within a distance of 5 kilometre from the coast include
Piram, Bhainsla near Kathiawar (Gujarat), Henery, Kainery, Butcher, Elephanta, Arnala
(in Bassein region), Bhatkal and Pigeoncock, etc. (north of Mangalore), Diu, Vaid, Nora,
Pirtan, Khariabet and Aliabet etc. (in Gujarat).
● The distant islands include the Lakshadweep, Minicoy, and Amindive. The
Lakshadweep islands are situated at a distance of 200-300 kilometre off the mainland
with an area of 108.78 square kilometers. Minicoy Island (4.3 square kilometre) lies
southwards.
Bay of Bengal Islands:
● The North and the Middle Andamans are highly dissected and densely forested and rise
to 730 metre in height. The South Andamans are largely flat lowlands.
● The Nicobar group of islands containing 19 islands is situated between 6°3'N and 9°3'N
latitudes. The Great Nicobar with an area of 862 square kilometers is the largest island
in the group. The little Nicobar, Katchall, Comerta, Trinket, Nankowry, Teressa and
Tillanchag are other important islands. The Barren and Narcondam islands situated
north of port Blair are volcanic islands.
Relevance: Experts have recently stated that the cyclones forming over Bay of Bengal are increasingly getting
stronger and larger, and as a consequence, their impact is being felt in wider areas.

Q83.
Answer: b
Explanation:

53
Vajiram & Ravi Prelims (2024)
PowerUp All India Mock Test - 01
● Tibetan Buddhism has 4 major schools: Nyingma (8th century), Kagyu (11th century), Sakya (1073) and
Gelug (1409). They all share a common goal of attaining enlightenment through the use of tantric
practices and spiritual development. Cham dance is used in all four schools of Tibetan Buddhism—
Nyingma, Sakya, Kagyu and Gelug—in their rituals as a sacred dance that is reflective of Buddhist tantric
practices.
○ Nyingma School: It is the oldest of the four, founded in the 8th century. It is also known as the
“ancient” school and is known for its teachings on Dzogchen, which is a path to enlightenment
that emphasises the natural state of the mind.
○ Kagyu school: It was founded in the early 11th century. It is known for its teachings on
Mahamudra, which is a path to enlightenment that emphasises the nature of the mind and the
practice of meditation.
○ Sakya school: It was founded in 1073. It is known for its teachings on Lamdre, which is a path to
enlightenment that emphasises the integration of sutra and tantra.
○ Gelug school: It was founded in 1409. It is also known as the “virtuous” school and is known for
its emphasis on the study of Buddhist philosophy and the use of reason and logic to understand
the nature of reality.
Therefore, option (b) is the correct answer.

Q84.
Answer: c
Explanation:
● The Chalukya temples are represented by the temples at Aihole and Badami. Among the seventy
temples found at Aihole, four are important.
○ Ladh Khan temple is a low, flat-roofed structure consisting of a pillared hall
○ Durga temple resembles a Buddha Chaitya
○ Huchimalligudi temple
○ Jain temple at Meguti
Therefore, option (c) is the correct answer.
Knowledge Box
● Among the temples at Badami, the Muktheeswara temple and the Melagutti
Sivalaya are notable for their architectural beauty. A group of four rock-cut temples
at Badami are marked by high workmanship. The walls and pillared halls are
adorned by beautiful images of gods and human beings.
● The second stage of the Chalukyan temples is represented by the temples at
Pattadakal. There are ten temples here, four in the northern style and the
remaining six in the Dravidian style.
○ The Papanatha temple is the most notable in the northern style.
○ The Sangamesvara temple and the Virupaksha temple are famous for their
Dravidian style.
○ The Virupaksha temple is built on the model of the Kailasanatha temple at
Kanchipuram. It was built by one of the queens of Vikramaditya II. Sculptors
brought from Kanchi were employed in its construction.
Relevance: Recently, two significant finds discovered in Ummeda village in Telangana, which are from the
Chalukyan period that have been carved on rock boulders close to the Kalabhairava Swamy temple.

54
Vajiram & Ravi Prelims (2024)
PowerUp All India Mock Test - 01
Q85.
Answer: c
Explanation:
● The Rajatarangini (River of Kings), composed by Kalhana in the 12th century, is the main source of
historical information for the early medieval history of the Kashmir region in India. The Rajatarangini
is an account of the royal dynasties that ruled the kingdom of Kashmir from its putative origins to the
poet’s own time. It narrates nearly two millennia of the ancient and early medieval history of the Kashmir
Valley.
Therefore, option (c) is the correct answer.

Q86.
Answer: c
Explanation:
● Shabda Kalpadruma: It was a Sanskrit dictionary written by Radhakanta Deb. He was an advocate of
both progressive and conservative ideas. He knew Sanskrit, Persian and Arabic and authored Shabda
Kalpadruma. So, pair 1 is correctly matched.
○ Radhakanta Deb took an active part in founding the Calcutta School Book Society, an organisation
that aimed at publishing and distributing textbooks in schools, in 1817, and the Calcutta School
Society, which focused on teaching methods and establishing new schools, in 1818.
○ He was the first President of the British Indian Association. He played a major role with David
Hare and Raja Ram Mohan Roy in the founding of Hindu College in Calcutta.
○ He was an active member of the Agricultural and Horticultural Society of India, which was
established in 1820.
● Sambad Prabhakar: It was a Bengali daily newspaper founded by Ishwar Chandra Gupta. It began as a
weekly newspaper in 1831 and became a daily eight years later in 1839. It was the first Bengali daily
newspaper. It was edited by some of the most prominent Bengali intellectuals of the time, including
Ishwar Chandra Gupta, Bankim Chandra Chattopadhyay and Rabindranath Tagore. So, pair 2 is correctly
matched.
● Darpan: It was the first Marathi-language daily newspaper. It was founded by Balshastri Jambhekar in
1832. Jambhekar's editorials often criticised the British colonial government and the Hindu orthodoxy.
In 1839, the Darpan was banned by the British Government. So, pair 3 is not correctly matched.
● Sambad Kaumudi: It was a Bengali weekly newspaper founded by Raja Ram Mohan Roy in 1821. It was
a progressive newspaper that advocated for social reform and religious tolerance. So, pair 4 is correctly
matched.
So, only three of the above pairs are correctly matched.
Therefore, option (c) is the correct answer.

Q87.
Answer: b
Explanation:
● Ahmedabad Mill Strike: In March 1918, Mahatma Gandhi intervened in a dispute between the cotton
mill owners of Ahmedabad and the workers over the issue of discontinuation of the plague bonus.
Gandhi led the mill workers of Ahmedabad in a strike against the mill owners who had refused to pay
them higher wages and exploited them severely. He rallied them by undertaking a fast and this united
all so firmly that the mill owners gave in on the fourth day of the fast and agreed to a 35 per cent wage
increase.

55
Vajiram & Ravi Prelims (2024)
PowerUp All India Mock Test - 01
● Rowlatt Act: It was passed in March 1919. On the recommendation of the Rowlatt committee the
Government in India prepared two draft bills to enable the government to check anti-state activities
and to empower the government with discretionary power to deal with political crime. The committee
had recommended that activists should be deported or imprisoned without trial for two years and that
even possession of seditious newspapers would be adequate evidence of guilt.
● Non-Cooperation Movement (NCM): During 1919-22, the British were opposed through two mass
movements—the Khilafat and Non-Cooperation. Though the two movements emerged from separate
issues, they adopted a common programme of action—that of non-violent non-cooperation. It was
formally launched on August 1, 1920. Later a special session of the Indian National Congress at Calcutta
approved the NCM programme till the Punjab and Khilafat wrongs were removed and Swaraj was
established.
● Second Moplah Revolt: It occurred between August 1921 and the end of 1922. It started as a resistance
against British colonial rule in the Malabar region of present-day Kerala, India. It later took a communal
colour as numerous acts of violence were reported and a series of persecutions were committed against
the Hindu landlords.
○ A hike in revenue demand and reduction of field size, coupled with the oppression of officials,
resulted in widespread peasant unrest among the Moplahs of Malabar. Twenty-two rebellions
took place between 1836 and 1854. None, however, proved successful.
So, the correct chronological order of the above events, starting from the earliest time: Ahmedabad mill strike
(1918) - Rowlatt Act (1919) - Non-Cooperation movement (1920) - Second Moplah revolt (1921).
Therefore, option (b) is the correct answer.

Q88.
Answer: c
Explanation:
● The Simon Commission published a two-volume report in May 1930. It proposed the abolition of
dyarchy and the establishment of representative government in the provinces which should be given
autonomy. It said that the governor should have discretionary power in relation to internal security and
administrative powers to protect the different communities. So, points 1 and 2 are correct.
● The number of members of the provincial legislative council should be increased. The report rejected
parliamentary responsibility at the centre. The governor-general was to have complete power to
appoint the members of the cabinet. And the Government of India would have complete control over
the high court. It also recommended that separate communal electorates be retained (and extended
such electorates to other communities) but only until tensions between Hindus and Muslims had died
down. There was to be no universal franchise. It accepted the idea of federalism but not in the near
future. It suggested that a Consultative Council of Greater India should be established which should
include representatives of both the British provinces as well as princely states. So, point 3 is correct and
point 4 is not correct.
● It suggested that the North-West Frontier Province (NWFP) and Baluchistan should get local legislatures,
and both NWFP and Balochistan should have the right to be represented at the centre. It recommended
that Sindh should be separated from Bombay, and Burma should be separated from India because it
was not a natural part of the Indian subcontinent. It also suggested that the Indian army should be
Indianised though British forces must be retained. So, point 5 is correct.
Therefore, option (c) is the correct answer.

56
Vajiram & Ravi Prelims (2024)
PowerUp All India Mock Test - 01
Q89.
Answer: a
Explanation:
Graph Networks for Materials Exploration
(GNoME):

● It is a state-of-the-art Graph Neural Network


(GNN) model where the input data for the
model takes the form of a graph that can
then be likened to connections between
atoms. It is an Artificial Intelligence tool
developed by Google DeepMind.
● GNoME was trained using “active learning”,
a technique to scale up a model first trained on a small specialised dataset. Developers can then
introduce new targets allowing machine learning to label new data with human assistance. This makes
the algorithm “well suited” to the science of discovering new materials, which requires searching for
patterns not found in the original dataset. It has significantly boosted the precision rate for predicting
material stability.
● Its performance was repeatedly checked using established computational techniques known as Density
Functional Theory (DFT).
Therefore, option (a) is the correct answer.

57
Vajiram & Ravi Prelims (2024)
PowerUp All India Mock Test - 01
Knowledge Box

● Density Functional Theory (DFT) is a Quantum-Mechanical (QM) method used in


chemistry and physics to calculate the electronic structure of atoms, molecules and
solids. It has been prevalent in computational solid-state physics.
● Active Learning is a special case of Supervised Machine Learning. This approach is used
to construct a high-performance classifier while keeping the size of the training dataset
to a minimum by actively selecting the valuable data points.
Relevance: Recently, GNoME has predicted the structures of millions of new materials and published a list of
the most stable ones.

Q90.
Answer: c
Explanation:
Tyndall effect: It is the phenomenon by virtue of which a beam of light gets scattered after striking the particles
that are present in its path. It is named after an Irish physicist John Tyndall. The phenomenon of the Tyndall
effect helps the path of light to be clearly visible. Examples of the Tyndall effect include the following.
● Visible rays of sunlight and blue colour of the sky: When the rays of the sunlight hit the dust particles
which are significantly larger in size than the wavelength of the light wave, all the colours contained by
the light wave get equally reflected making the path of the light clearly visible.
○ However, when the sunlight strikes particles of gases like oxygen and nitrogen that are smaller
in size than the wavelength of the sunlight, the colours having smaller wavelengths, i.e., blue and
violet, get much more scattered. Our senses tend to respond to the blue colour more than the
violet. Hence, the sky appears blue. So, point 1 is correct.
● Scattering of car light in fog: The visible layer of water vapours and ice crystals that is present above the
surface of the earth is called fog.
○ During foggy weather, the light rays emitted from the headlight of a car strike the particles
contained by the fog and get scattered away, thereby allowing the observer to trace the path
taken by the light to travel. So, point 2 is correct.
● Light shined through milk: The Tyndall effect is best demonstrated by colloidal solutions. When laser
light is made to shine through a container that is filled with a mixture of milk and water, its path is clearly
visible as the particles of the colloid are larger in size than the wavelength of the light.
○ Since particles such as molecules of sugar or sodium ions or chloride ions in sugar solution are
too small to scatter light, a beam of light passing through such a solution is not scattered. So,
point 3 is not correct.
● Passage of sunlight through the canopy of a dense forest: Tyndall effect can also be observed when a
fine beam of light enters a room through a small hole. This happens due to the scattering of light by the
particles of dust and smoke in the air. It can be observed when sunlight passes through the canopy of a
dense forest. In the forest, mist contains tiny droplets of water, which act as particles of colloid dispersed
in the air. So, point 4 is correct.
● Smoke from motorcycles: The burning of the engine oil in the 2 stroke or 4 stroke engine of a motorcycle
releases particles of smoke in the air that are bigger in size as compared to the wavelength of the light.
Hence, the smoke coming out of such engines appears to be a little blue near the areas from where the
smoke emerges out. It thus exhibits a Tyndall effect. So, point 5 is correct.
So, only four of the above exhibit a Tyndall effect.
Therefore, option (c) is the correct answer.
58
Vajiram & Ravi Prelims (2024)
PowerUp All India Mock Test - 01
Relevance: Recently, a Tyndall effect landscape was seen over Wuhu, East China's Anhui Province.

Q91.
Answer: d
Explanation:
● According to researchers at the University of Tokyo, the omicron subvariant JN.1. is likely to become the
dominant lineage of the SARS-CoV-2 virus worldwide. The subvariant has a mutation in its spike protein,
L455S, also called a “FLip” mutation.
● L455S and L455F are called “FLip” mutations because they switch the positions of amino acids F and L
on the spike protein, increasing their transmissibility. Another example of the “FLip” mutation is F456L.
Therefore, option (d) is the correct answer.
Relevance: According to researchers, FLip mutations are responsible for recent surge in COVID cases.

Q92.
Answer: d
Explanation:
● eSoil is a low-power bioelectronic growth substrate that can electrically stimulate the root system and
growth environment of plants. This novel substrate is not only environmentally friendly, but also offers
a low energy, safe alternative to previous methods that required high voltage and non-biodegradable
materials.
● eSoil uses low energy and minimises resource consumption. Its active material is an organic mixed-ionic
electronic conductor.
○ The advantage of eSoil is that it has very low energy consumption and not high-voltage danger.
The study showed that barley seedlings can be cultivated using hydroponics and that they have
a better growth rate due to electrical stimulation. So, point 1 is correct.
○ It stimulates root systems and helps barley seedlings grow an average of 50 per cent more. The
low-power bioelectronic scaffolding can be used for soilless cultivation methods like
hydroponics. So, point 2 is correct.
○ Hydroponics refers to the technique of growing plants without soil using a water-based nutrient
solution, an aggregate substrate, or growing media like coconut coir or perlite. It uses closed
systems that recirculate water to make sure each seedling gets exactly the nutrients it needs.
This means that very little water is needed and that all nutrients remain within the system. So,
point 3 is correct.
○ eSoil can help in areas with little arable land and harsh environmental conditions. The seedlings
process nitrogen more effectively, but it’s not clear yet how the electrical stimulation impacts
this process. So, point 4 is correct.
○ Hydroponic systems often use mineral wool as a cultivation substrate in hydroponics. But these
wools are often non-biodegradable and are produced using energy-intensive processes. eSoil is
made of cellulose, a biopolymer mixed with a conductive polymer called PEDOT.
Therefore, option (d) is the correct answer.
Relevance: Recently, researchers have developed a new electronic soil that was found to increase the growth
of barley seedlings by 50 per cent.

Q93.
Answer: c
Explanation:

59
Vajiram & Ravi Prelims (2024)
PowerUp All India Mock Test - 01
● Radiometric dating is a technique used to determine the age of objects like wooden artefacts, rocks,
or fossils by analysing the presence of radioactive isotopes within them. It relies on comparing the
isotope's concentration in the sample to its known abundance on Earth, along with its established half-
life (decay rate), allowing for the calculation of the sample's age. This dating method is particularly
valuable for ancient objects due to the gradual decay of radioactive materials, and it is based on the
natural decay process of radioactive elements. So, statement 1 is correct.
○ It’s a very common method used mostly by archaeologists because it can only date relatively
recent materials. Radiocarbon dating is not suitable for dating anything older than around
50,000 years.
● Uranium-Lead dating is a reliable method for dating rocks that are millions or even billions of years old.
It involves measuring the ratio of uranium to lead in a rock sample to ascertain the time when the rock
solidified. This approach is instrumental in understanding Earth's formation and determining the age
of geological periods, including the era of dinosaurs. So, statement 2 is correct.
● Potassium-Argon dating is vital for determining the formation time of volcanic rocks and exploring Earth's
volcanic history. Using the radioactive isotopes potassium-40 and argon-40, the ratio of radioactive
potassium, argon, and calcium is measured and compared over time. So, statement 3 is correct.
So, all three of the above statements are correct.
Therefore, option (c) is the correct answer.
Relevance: A recent study has proposed using calcium-41 as a new method for radiometric dating.

Q94.
Answer: d
Explanation:
Biomining:
● According to the Central Pollution Control Board (CPCB) guidelines, biomining is the scientific process
of excavation, treatment, segregation and gainful utilisation of aged municipal solid waste lying in
dumpsites typically referred to as legacy waste.
● It is a process of treating garbage or waste with bio-organisms or natural elements like air and sunlight.
Over time, the biodegradable component of the waste gets decomposed through the natural process
and the remaining part i.e. non-biodegradable material is then dealt with separately. It extracts metals
of economic interest from rock ores or mine waste.
● It comprises four steps: excavation of legacy waste, stabilising the waste using bioremediation,
segregation of excavated waste and then sustainable management and its safe disposal.
● The most common processes used in biomining are:
○ Heap leaching: Freshly mined material is moved directly into heaps that are then bio-leached.
So, point 1 is correct.
○ Dump leaching: Low-value ore or waste rock is placed in a sealed pit and then bio-leached to
remove more of the valuable metals from the waste pile. So, point 2 is correct.
○ Biological oxidation: It is the combination of oxidation-reduction transformations of substances
in living organisms. In bio-oxidation, on the other hand, gold is predominantly unlocked from
refractory ores in large-scale stirred-tank bio-oxidation arrangements for further processing
steps. So, point 3 is correct.
○ Agitated leaching: Crushed rocks are placed into a large vat that is shaken to distribute the
microbes and material evenly and speed up the bioleaching process. So, point 4 is correct.

60
Vajiram & Ravi Prelims (2024)
PowerUp All India Mock Test - 01
● Most current biomining operations target valuable metals like copper, uranium, nickel, and gold that are
commonly found in sulfidic (sulphur-bearing) minerals. Microbes are especially good at oxidising sulfidic
minerals, converting metals like iron and copper into forms that can dissolve more easily.
Therefore, option (d) is the correct answer.
Knowledge Box

● The Solid Waste Management Rules, 2016 now mandate to bio-mine legacy waste
instead of 'capping' which means covering the waste with soil. These rules focus on the
segregation of waste at the source, the responsibility of the manufacturer to dispose of
sanitary and packaging wastes and user fees for collection, disposal and processing
from the bulk generator.
○ Legacy wastes are those that have been collected and kept for years at some
barren land or a place dedicated to Landfill.
○ They are the result of uncontrolled and continuous dumping of municipal solid
waste.
○ In India, legacy wastes have predominantly 4 fractions of waste: fine soil/sand-
like material, scrap polymeric and combustible materials, stones (greater than
20 millimetres in size) and miscellaneous items.
Advantages of biomining:
● Zero emissions and leaving near-zero residues.
● Reduction in soil pollution, soil contamination and groundwater pollution.
● Reduction in greenhouse gas emissions and no energy is required for the process.
● Further, the reclaimed land can be used for other development purposes.
● Biomining is an eco-friendly method that allows us to recycle resources by extracting
useful components from waste such as metal, compost in fertiliser etc.
Disadvantages of biomining:
● It is restricted to only biodegradable compounds and takes time to show results.
● One of the significant environmental risks is the risk of leakage and treatment of the
acidic, metal-rich solution created by the microbes.
Relevance: The Dindigul Corporation has commenced the process of removing fresh solid waste, segregated
waste and legacy waste by using biomining techniques.

Q95.
Answer: b
Explanation:
● The Group of Twenty (G20) is a grouping which brings together the countries with the largest economies
in the world. The member states meet annually to discuss economic, political and social initiatives. The
group defines itself as the main forum for international economic cooperation.
● Mission LiFE (Lifestyle for Environment): It is an India-led global mass movement to nudge individual
and community action to protect and preserve the environment. It was launched at the 26th session of
the Conference of the Parties (COP26) to the United Nations Framework Convention on Climate Change
(UNFCCC) held in Glasgow, United Kingdom (2021). So, point 1 is not correct.
● Global Biofuels Alliance: It was launched with the support of G20 countries and other organisations on
the sidelines of the latest G20 meeting under India’s presidency in 2023. It aims to ensure energy
security, affordability and accessibility for the future. It also aims to facilitate global collaboration,
supporting the development and deployment of sustainable biofuels. So, point 2 is correct.
61
Vajiram & Ravi Prelims (2024)
PowerUp All India Mock Test - 01
● Green Development Pact: It was launched at the G20 Summit in 2023 during India’s Presidency. It
recognises that the present and future generations can be prosperous only if current development and
other policy choices and actions resolve environmentally sustainable practices and inclusive economic
growth. It recognises that global warming can be limited to 1.5 degrees Celsius through reductions in
global greenhouse gas emissions by 43 per cent by 2030, compared to the 2019 levels. So, point 3 is
correct.
● Green Grids Initiative (GGI): It aims to accelerate the construction of the new infrastructure needed
for a world powered by renewable energy. It includes smart grids connecting millions of solar panels
and charging points for electric vehicles and micro-grids for rural communities to ensure resilience during
extreme weather. In 2021, the GGI was launched in partnership with India's One Sun One World One
Grid (OSOWOG) during the UNFCCC COP26 World Leaders' Summit. So, point 4 is not correct.
● India-Middle East-Europe Economic Corridor (IMEC): It was launched at the G20 Summit in 2023 during
India’s Presidency. It is a multi-billion connectivity scheme linking India and Europe. India’s co-signatories
in this endeavour are the United Arab Emirates (UAE), Saudi Arabia, the United States, Germany, France
and the European Union (EU). So, point 5 is correct.
So, only three of the above were launched as part of India’s G20 Presidency.
Therefore, option (b) is the correct answer.

Knowledge Box

● Other new initiatives that were launched during India’s G20 Presidency were:
○ Disaster Risk Reduction Working Group
○ Startup20 Engagement Group
○ Chief Scientific Advisors Roundtable
○ G20 Conference on Crime and Security in the Age of Non-Fungible Tokens,
Artificial Intelligence and Metaverse
○ New Working Group on Empowerment of Women
○ Cooperation in counternarcotics
○ Cooperation in traditional medicines
○ Millet-research Initiative
Relevance: Recently, the 18th Summit of G20 took place in New Delhi in 2023 under India’s Presidency.

Q96.
Answer: b
Explanation:
● Malawi was recently in news due to a huge devaluation of the currency as it secured a loan from the
International Monetary Fund (IMF) to boost its ailing economy. So, pair 1 is not correctly matched.
● Nigeria: Ethnic attacks led by Bandit militias have broken out in Central Nigeria’s Plateau state leading
to the death and injury of several people. Plateau is one of several ethnically and religiously diverse
hinterland states known as Nigeria's Middle Belt, where inter-communal conflict has claimed hundreds
of lives in recent years. So, pair 2 is correctly matched.
● Niger: Recently, a military coup took place in Niger in which several factions of the armed forces seized
power and suspended the constitution. The coup reflects mounting security problems and internal
power struggles. It is part of a trend of military coups in the region since 2019, indicating weakening
Western influence and increased engagement by Russia. So, pair 3 is not correctly matched.

62
Vajiram & Ravi Prelims (2024)
PowerUp All India Mock Test - 01
● Sudan: Recently, violent clashes broke out between the army and a powerful paramilitary force for
absolute control of Sudan. With 3,000 Indians stuck in Sudan, the Government of India undertook
Operation Kaveri to evacuate them. So, pair 4 is correctly matched.
So, only two of the above pairs are correctly matched.
Therefore, option (b) is the correct answer.
Relevance: Armed militant groups in Nigeria have launched a series of brutal attacks in central Nigeria state of
Plateau.

Q97.
Answer: c
Explanation:
● The Tropic of Cancer is an imaginary line that lies at an angle of 23.50 degrees north from the Equator.
● This latitude marks the last point where the Sun appears
directly overhead at its zenith (its highest point) on the
June or Summer Solstice (also known as the Midsummer
or Festival Solstice).
● It runs across three continents and through 16 countries,
these countries being Mexico, the Bahamas, Bangladesh,
Myanmar, Niger, Oman, Mauritania, Mali, Algeria, Libya,
Egypt, Saudi Arabia, the United Arab Emirates, India,
Taiwan and China.
● It also runs through six major bodies of water, these being the Indian Ocean, Atlantic Ocean, Pacific
Ocean, Taiwan Strait, Red Sea, Philippine Sea and Gulf of Mexico. So, points 1, 2, 3 and 4 are correct.
● It does not pass through the Mediterranean Sea. So, point 5 is not correct.
So, the Tropic of Cancer passes through only four of the above water bodies.
Therefore, option (c) is the correct answer.

Knowledge Box

● Zero Shadow is a phenomenon when the Sun is exactly overhead and the shadows of
symmetrical and vertical objects vanishes.
● This happens for locations between the tropics and is caused by the northern and
southern motion of the Sun during the course of a year.
Relevance: Zero Shadow Day was observed in Bengaluru at 12.17 pm on April 25, 2023, where the Sun shines
directly on the Tropic of Cancer.

Q98.
Answer: c
Explanation:
In India, certain intelligence and security organisations specified in the Second Schedule of the Right to
Information Act, 2005, are exempted from providing information excepting the information pertaining to the
allegations of corruption and human rights violations.
● The Indian Computer Emergency Response Team (CERT-In) is the national nodal agency for responding
to computer security incidents as and when they occur. Recently, the Centre has used its powers given
under Section 24(2) of the RTI Act to exempt CERT-In from the purview of the transparency law. So,
point 1 is correct.
63
Vajiram & Ravi Prelims (2024)
PowerUp All India Mock Test - 01
● There are 26 other intelligence and security organisations that are already exempted from the Act,
which includes - Intelligence Bureau; Research and Analysis Wing including its technical wing namely;
Directorate of Enforcement; Narcotics Control Bureau; Financial Intelligence Unit; Special Protection
Group; Defence Research and Development Organisation; Central Bureau of Investigation; National
investigation Agency; and many more. So, points 2, 3 and 5 are correct.
● The RTI Act, 2005 extends to the whole of India. All bodies, which are constituted under the Constitution
or under any law or under any Government notification or all bodies, including NGOs, which are owned,
controlled or substantially financed by the Government are covered. The Indian Space Research
Organization (ISRO) is not exempted under RTI Act. So, point 4 is not correct.
So, only four of the above are exempted from the purview of the Right to Information Act, 2005 in India.
Therefore, option (c) is the correct answer.
Relevance: Recently, the Centre has exempted the Indian Computer Emergency Response Team (CERT-In) from
the purview of the Right to Information Act, 2005.

Q99.
Answer: c
Explanation:
● Legion of Honour: It is the highest civilian honour of France, conferred on both military and civilian
orders. It was established by Napoleon Bonaparte. It is typically given to individuals who have
demonstrated remarkable merit in military or civilian services, which also includes foreigners who have
supported causes championed by France. The Prime Minister of India was awarded the ‘Grand Cross of
the Legion of Honour’, in 2023. So, pair 1 is correctly matched.
● Champions of the Earth: This is the United Nations highest environmental honour, recognizes
outstanding leaders from government, civil society and the private sector whose actions have a
transformative impact on the environment. It also acknowledges individuals or organisations driving
technological advancements for significant environmental benefits. So, pair 2 is correctly matched.
● Global Goalkeeper Award: This is an annual recognition given by the Goalkeepers in partnership with
the Bill & Melinda Gates Foundation to remarkable individuals taking action to help achieve the Global
Goals by 2030 under the Project Everyone, a United Nations Global Partner for Sustainable
Development Goals (SDGs) Advocacy and Outreach. So, pair 3 is correctly matched.
So, all three of the above pairs are correctly matched.
Therefore, option (c) is the correct answer.
Relevance: Recently, the Prime Minister of India was awarded France's highest civilian and military honour
Legion of Honour.

Q100.
Answer: c
Explanation:
● National Science Day (NSD): It is celebrated every year on 28 February to commemorate the discovery
of the ‘Raman Effect’. So, statement 1 is correct.
● The Government of India designated 28 February as National Science Day (NSD) in 1986. On this day Sir
C.V. Raman announced the discovery of the 'Raman Effect' for which he was awarded the Nobel Prize
in 1930. On this occasion, theme-based science communication activities are carried out all over the
country. So, statement 2 is not correct.

64
Vajiram & Ravi Prelims (2024)
PowerUp All India Mock Test - 01
○ The Raman effect involves scattering of light by molecules of gases, liquids, or solids. The Raman
effect consists of the appearance of extra spectral lines near the wavelength of the incident light.
The Raman lines in the scattered light are weaker than the light at the original wavelength.
○ The Aryabhata spacecraft, named after the famous Indian astronomer, was India's first satellite;
it was completely designed and fabricated in India and launched by a Soviet Kosmos-3M rocket
from Kapustin Yar on April 19, 1975.
So, statement-I is correct but Statement-II is incorrect.
Therefore, option (c) is the correct answer.
Relevance: Recently the birth anniversary of Sir C.V. Raman was celebrated in various parts of the country.

65
Vajiram & Ravi Prelims (2024)
PowerUp All India Mock Test - 01

You might also like